Sie sind auf Seite 1von 81

Problems and Solutions in

REAL AND COMPLEX ANALYSIS


William J. DeMeo
July 9, 2010
c _William J. DeMeo. All rights reserved. This document may be copied for personal use. Permission to reproduce
this document for other purposes may be obtained by emailing the author at williamdemeo@gmail.com.
Abstract
The pages that follow contain unofcial solutions to problems appearing on the comprehensive exams in
analysis given by the Mathematics Department at the University of Hawaii over the period from 1991 to 2007. I have
done my best to ensure that the solutions are clear and correct, and that the level of rigor is at least as high as that
expected of students taking the ph.d. exams. In solving many of these problems, I beneted enormously from the
wisdom and guidance of professors Tom Ramsey and Wayne Smith. Of course, some typos and mathematical errors
surely remain, for which I am solely responsible. Nonetheless, I hope this document will be of some use to you as
you prepare to take the comprehensive exams. Please email comments, suggestions, and corrections to
williamdemeo@gmail.com.
Contents
1 Real Analysis 3
1.1 1991 November 21 . . . . . . . . . . . . . . . . . . . . . . . . . . . . . . . . . . . . . . . . . . . . 3
1.2 1994 November 16 . . . . . . . . . . . . . . . . . . . . . . . . . . . . . . . . . . . . . . . . . . . . 10
1.3 1998 April 3 . . . . . . . . . . . . . . . . . . . . . . . . . . . . . . . . . . . . . . . . . . . . . . . . 12
1.4 2000 November 17 . . . . . . . . . . . . . . . . . . . . . . . . . . . . . . . . . . . . . . . . . . . . 18
1.5 2001 November 26 . . . . . . . . . . . . . . . . . . . . . . . . . . . . . . . . . . . . . . . . . . . . 20
1.6 2004 April 19 . . . . . . . . . . . . . . . . . . . . . . . . . . . . . . . . . . . . . . . . . . . . . . . 27
1.7 2007 November 16 . . . . . . . . . . . . . . . . . . . . . . . . . . . . . . . . . . . . . . . . . . . . 32
2 Complex Analysis 38
2.1 1989 April . . . . . . . . . . . . . . . . . . . . . . . . . . . . . . . . . . . . . . . . . . . . . . . . . 39
2.2 1991 November 21 . . . . . . . . . . . . . . . . . . . . . . . . . . . . . . . . . . . . . . . . . . . . 43
2.3 1995 April 10 . . . . . . . . . . . . . . . . . . . . . . . . . . . . . . . . . . . . . . . . . . . . . . . 48
2.4 2001 November 26 . . . . . . . . . . . . . . . . . . . . . . . . . . . . . . . . . . . . . . . . . . . . 52
2.5 2004 April 19 . . . . . . . . . . . . . . . . . . . . . . . . . . . . . . . . . . . . . . . . . . . . . . . 59
2.6 2006 November 13 . . . . . . . . . . . . . . . . . . . . . . . . . . . . . . . . . . . . . . . . . . . . 60
2.7 2007 April 16 . . . . . . . . . . . . . . . . . . . . . . . . . . . . . . . . . . . . . . . . . . . . . . . 64
2.8 2007 November 16 . . . . . . . . . . . . . . . . . . . . . . . . . . . . . . . . . . . . . . . . . . . . 68
2.9 Some problems of a certain type . . . . . . . . . . . . . . . . . . . . . . . . . . . . . . . . . . . . . 69
1
A Miscellaneous Denitions and Theorems 71
A.1 Real Analysis . . . . . . . . . . . . . . . . . . . . . . . . . . . . . . . . . . . . . . . . . . . . . . . 71
A.1.1 Metric Spaces . . . . . . . . . . . . . . . . . . . . . . . . . . . . . . . . . . . . . . . . . . . 71
A.1.2 Measurable Functions . . . . . . . . . . . . . . . . . . . . . . . . . . . . . . . . . . . . . . 71
A.1.3 Integration . . . . . . . . . . . . . . . . . . . . . . . . . . . . . . . . . . . . . . . . . . . . 72
A.1.4 Approximating Integrable Functions
1
. . . . . . . . . . . . . . . . . . . . . . . . . . . . . . 73
A.1.5 Absolute Continuity of Measures . . . . . . . . . . . . . . . . . . . . . . . . . . . . . . . . 74
A.1.6 Absolute Continuity of Functions . . . . . . . . . . . . . . . . . . . . . . . . . . . . . . . . 75
A.1.7 Product Measures and the Fubini-Tonelli Theorem . . . . . . . . . . . . . . . . . . . . . . . 75
A.2 Complex Analysis . . . . . . . . . . . . . . . . . . . . . . . . . . . . . . . . . . . . . . . . . . . . . 77
A.2.1 Cauchys Theorem
2
. . . . . . . . . . . . . . . . . . . . . . . . . . . . . . . . . . . . . . . . 77
A.2.2 Maximum Modulus Theorems . . . . . . . . . . . . . . . . . . . . . . . . . . . . . . . . . . 78
B List of Symbols 79
2
1 REAL ANALYSIS
1 Real Analysis
1.1 1991 November 21
1. (a) Let f
n
be a sequence of continuous, real valued functions on [0, 1] which converges uniformly to f. Prove that
lim
n
f
n
(x
n
) = f(1/2) for any sequence x
n
which converges to 1/2.
(b) Must the conclusion still hold if the convergence is only point-wise? Explain.
Solution: (a) Let x
n
be a sequence in [0, 1] with x
n
1/2 as n . Fix > 0 and let N
0
N be such
that n N
0
implies [f
n
(x) f(x)[ < /2, for all x [0, 1]. Let > 0 be such that [f(x) f(y)[ < /2,
for all x, y [0, 1] with [x y[ < . Finally, let N
1
N be such that n N
1
implies [x
n
1/2[ < . Then
n maxN
0
, N
1
implies [f
n
(x
n
) f(1/2)[ [f
n
(x
n
) f(x
n
)[ +[f(x
n
) f(1/2)[ < /2 +/2 = . .
(b) Suppose the convergence is only point-wise. Then the conclusion is false, as the following counterexample
demonstrates:
Dene f
n
(x) to be the function
f(x) =
_

_
0, if 0 x <
1
2

1
2n
,
2nx (n 1), if
1
2

1
2n
x <
1
2
,
1, if
1
2
x 1.
(1)
That is, f
n
(x) is constantly zero for x less than
1
2

1
2n
, then it increases linearly until it reaches one at x = 1/2,
and then it remains constantly one for x bigger than 1/2. Now dene the sequence x
n
=
1
2

1
n
. Then f
n
(x
n
) = 0
for all n N and x
n
1/2, while the sequence f
n
approaches the characteristic function f
[
1
2
,1]
which is one
on [
1
2
, 1] and zero elsewhere. Therefore, f(1/2) = 1 ,= 0 = lim
n
f
n
(x
n
). .
2. Let f : R R be differentiable and assume there is no x R such that f(x) = f

(x) = 0. Show that


S = x[ 0 x 1, f(x) = 0 is nite.
Solution: Consider f
1
(0). Since 0 is closed and f continuous, f
1
(0) is closed. Therefore S = [0, 1]
f
1
(0) is a closed and bounded subset of R. Hence, S is compact. Assume, by way of contradiction, that S is
innite. Then (by theorem A.1) there is a limit point x S; i.e., there is a sequence x
n
of distinct points in S
which converges to x. Also, as all points are in S, f(x
n
) = f(x) = 0 for all n N.
We now show that f

(x) = 0, which will give us our desired contradiction. Since [x


n
x[ 0, we can write the
derivative of f as follows:
f

(x) = lim
n
f(x + (x
n
x)) f(x)
x
n
x
= lim
n
f(x
n
) f(x)
x
n
x
= 0.
The last equality holds since f(x) = f(x
n
) = 0 holds for all n N. .
3. If (X, , ) is a measure space and if f is integrable, show that for every > 0 there is E such that
(E) < and
_
X\E
[f[ d < .
3
1.1 1991 November 21 1 REAL ANALYSIS
Solution: For n = 1, 2, . . ., dene
A
n
= x X : 1/n [f(x)[ < n.
Clearly,
A
1
A
2
A

_
n=1
A
n
and each A
n
is measurable (why?).
3
Next, dene
A
0
= x X : f(x) = 0 and A

= x X : [f(x)[ = .
Then X = A
0
A A

is a disjoint union, and


_
X
[f[ =
_
A0
[f[ +
_
A
[f[ +
_
A
[f[ =
_
A
[f[. (2)
The rst term in the middle expression is zero since f is zero on A
0
, and the third term is zero since f L
1
()
implies (A

) = 0. To prove the result, then, we must nd a measurable set E such that


_
A\E
[f[ < , and
(E) < .
Dene f
n
= [f[
An
. Then f
n
is a sequence of non-negative measurable functions and, for each x X,
lim
n
f
n
(x) = [f(x)[
A
(x). Since A
n
A
n+1
, we have 0 f
1
(x) f
2
(x) , so the monotone
convergence theorem
4
implies
_
X
f
n

_
A
[f[, and, by (2),
lim
n
_
An
[f[ d =
_
A
[f[ d =
_
X
[f[ d.
Therefore, there is some N > 0 for which
_
X\A
N
[f[ d < .
Finally, note that 1/N [f[ < N on A
N
, so
(A
N
) N
_
A
N
[f[ d N
_
X
[f[ d < .
Therefore, the set E = A
N
meets the given criteria. .
4.
5
If (X, , ) is a measure space, f is a non-negative measurable function, and (E) =
_
E
f d, show that is a
measure.
Solution: Clearly (E) = 0 (E) = 0. Therefore, () = () = 0. In particular is not identically innity,
so we need only check countable additivity. Let E
1
, E
2
, . . . be a countable collection of disjoint measurable sets.
3
Answer: f is measurable and x |x| is continuous, so g = |f| is measurable. Therefore, An = g
1
([1/n, n)) is measurable (theorem A.2).
4
Alternatively, we could have cited the dominated convergence theorem here since fn(x) |f(x)| (x X; n = 1, 2, . . .).
5
See also: Rudin [8], chapter 1. Thanks to Matt Chasse for pointing out a mistake in my original solution to this problem. I believe the solution
given here is correct, but the skeptical reader is encouraged to consult Rudin.
4
1.1 1991 November 21 1 REAL ANALYSIS
Then,
(
n
E
n
) =
_

n
En
f d =
_
f

n
En
d
=
_

n=1
f
En
d ( the E
n
are disjoint)
=

n=1
_
f
En
d ( f
En
0, n = 1, 2, . . .)
=

n=1
(E
n
).
The penultimate equality follows from the monotone convergence theorem applied to the sequence of non-negative
measurable functions g
m
=

m
n=1
f
En
(m = 1, 2, . . .). (See also: April 98, problem A.3.) .
5. Suppose f is a bounded, real valued function on [0, 1]. Show that f is Lebesgue measurable if and only if
sup
_
dm = inf
_
dm
where m is Lebesgue measure on [0, 1], and and range over all simple functions, f .
Solution: This is proposition 4.3 of Royden, 3ed. [6].
6.
6
If f is Lebesgue integrable on [0, 1] and > 0, show that there is > 0 such that for all measurable sets E [0, 1]
with m(E) < ,

_
E
f dm

< .
Solution: This problem appears so often, I think its worth giving two different proofs. The rst relies on the
frequently useful technique, employed in problem 3, in which the domain is written as a union of the nested sets
A
n
= x X : 1/n [f(x)[ < n. The second is a shorter proof, but it relies on a result about absolute
continuity of measures, which is almost equivalent to the original problem statement. I recommend that you learn
the rst proof. The second proof is also worth studying, however, as it connects this result to the analogous result
about absolutely continuous measures.
Proof 1: Let A
n
, n = 1, 2, . . . be the sequence of measurable sets dened in problem 3. That is,
A
n
= x X : 1/n [f(x)[ < n.
Here, X = [0, 1]. As we saw in problem 3,
lim
n
_
An
[f[ dm =
_
A
[f[ dm =
_
X
[f[ dm.
6
See also: April 92 (4), November 97 (6), April 03 (4).
5
1.1 1991 November 21 1 REAL ANALYSIS
Let n > 0 be such that
_
X\An
[f[ dm < /2.
Dene = (2n)
1
, and suppose E [0, 1] is a measurable set with mE < . We must show [
_
E
f dm[ < .

_
E
f dm

_
E
[f[ dm
=
_
(X\An)E
[f[ dm+
_
AnE
[f[ dm

_
X\An
[f[ dm+
_
AnE
[f[ dm
< /2 +nm(A
n
E)
< /2 +n

2n
= .
The penultimate inequality holds because [f[ < n on A
n
. .
Proof 2:
7
This proof relies on the following lemma about absolute continuity of measures:
Lemma 1.1 Let be a nite signed measure and a positive measure on a measurable space (X, M). Then
if and only if for every > 0 there is a > 0 such that [(E)[ < whenever (E) < .
The signed measure dened by
(E) =
_
E
f d
is nite iff
8
f L
1
(). It is also clearly absolutely continuous with respect to . Therefore, lemma 1.1 can be
applied to the real and imaginary parts of any complex-valued f L
1
(). It follows that, for every > 0, there is
a > 0 such that
[(E)[ =

_
E
f d

< , whenever (E) < .


.
7.
9
Suppose f is a bounded, real valued, measurable function on [0, 1] such that
_
x
n
f dm = 0 for n = 0, 1, 2, . . .,
with m Lebesgue measure. Show that f(x) = 0 a.e.
Solution: Fix an arbitrary continuous function on [0, 1], say, C[0, 1]. By the Stone-Weierstrass theorem, there
is a sequence p
n
of polynomials such that | p
n
|

0 as n . Then, since all functions involved are


7
See also Folland [4], page 89.
8
For what follows we only need that is nite if f is integrable, but the converse is also true.
9
This question appears very often in varying forms of difculty. cf. November 92 (7b, very easy version), November 96 (B2, very easy),
November 91 (this question, easy), April 92 (6, moderate), November 95 (6, hardimpossible?). I have yet to solve the November 95 version.
One attempted solution (which I think is the one given in the black notebook), seems to assume f L
1
, but that assumption makes the problem
even easier than the others.
6
1.1 1991 November 21 1 REAL ANALYSIS
integrable,

_
f

_
f( p
n
+p
n
)

_
[f[[ p
n
[ +

_
fp
n

|f|
1
| p
n
|

_
fp
n

= |f|
1
| p
n
|

. (3)
The last equality holds since
_
x
n
f = 0 for all n = 0, 1, 2, . . ., which implies that
_
fp
n
= 0 for all polynomials
p
n
. Finally, note that |f|
1
< , since f is bounded and Lebesgue measurable on the bounded interval [0, 1].
Therefore, the right-hand side of (3) tends to zero as n tends to innity. Since the left-hand side of (3) is independent
of n, we have thus shown that
_
f = 0 for any C[0, 1].
Now, since C[0, 1] is dense in L
1
[0, 1], let
n
C[0, 1] satisfy |
n
f|
1
0 as n . Then
0
_
f
2
=

_
f(f
n
+
n
)

_
[f[[f
n
[ +

_
f
n

.
The second term on the right is zero by what we proved above. Therefore, if M is the bound on [f[, we have
0
_
f
2
M|f
n
|
1
0. As
_
f
2
is independent of n, we have
_
f
2
= 0. Since f
2
0, this implies
f
2
= 0 a.e., hence f = 0 a.e. .
Alternative Solution: Quinn Culver suggests shortening the proof by using the fact that polynomials are dense in
L
1
[0, 1]. Simply start from the line, Now, since C[0, 1] is dense in L
1
[0, 1], let
n
C[0, 1] satisfy... but
instead write, Since Pol[0, 1] is dense in L
1
[0, 1], let
n
Pol[0, 1] satisfy... This is a nice observation and
disposes of the problem quickly and efciently. However, I have left the original, somewhat clumsy proof intact
because it provides a nice demonstration of the Stone-Weierstrass theorem (which appears on the exam syllabus),
and because everyone should know how to apply this fundamental theorem to problems of this sort.
8.
10
If and are nite measures on the measurable space (X, ), show that there is a nonnegative measurable
function f on X such that for all E in ,
_
E
(1 f) d =
_
E
f d. (4)
Solution: Theres an assumption missing here: and must be positive measures.
11
In fact, one can prove the
result is false without this assumption. So assume and are nite positive measures on the measurable space
(X, ). By the linearity property of the integral, and since (E) =
_
E
d, we have
_
E
(1 f) d =
_
E
d
_
E
f d = (E)
_
E
f d.
Therefore, (4) is equivalent to
(E) =
_
E
f d +
_
E
f d =
_
E
f d( +) ( E ) (5)
10
See also: November 97 (7).
11
Note that a measure is called positive when it is, in fact, nonnegative; that is, E 0 for all E .
7
1.1 1991 November 21 1 REAL ANALYSIS
so this is what we will prove. The Radon-Nikodym theorem (A.12) says, if m are -nite positive measures
on a -algebra , then there is a unique g L
1
(dm) such that
(E) =
_
E
g dm, E .
In the present case, + (since the measures are positive), so the theorem provides an f L
1
( + ) such
that
(E) =
_
E
f d( +) E ,
which proves (5). .
9.
12
If f and g are integrable functions on (X, o, ) and (Y, T , ), respectively, and F(x, y) = f(x) g(y), show that
F is integrable on X Y and
_
F d( ) =
_
f d
_
g d.
Solution:
13
To show F(x, y) = f(x)g(y) is integrable, an important (but often overlooked) rst step is to prove
that F(x, y) = f(x)g(y) is (o T )-measurable. Dene : X Y RR by (x, y) = (f(x), g(y)), and let
: R R R be the continuous function (s, t) = st. Then,
F(x, y) = f(x)g(y) = ( )(x, y).
Theorem A.2 states that a continuous function of a measurable function is measurable. Therefore, if we can show
that (x, y) is an (o T )-measurable function fromXY into RR, then it will follow that F(x, y) is (o T )-
measurable. To show is measurable, let R be an open rectangle in R R. Then R = AB for some open sets
A and B in R, and

1
(R) =
1
(AB)
= (x, y) : f(x) A, g(y) B
= (x, y) : f(x) A (x, y) : g(y) B
= (f
1
(A) Y ) (X g
1
(B))
= f
1
(A) g
1
(B).
Now, f
1
(A) o and g
1
(B) T , since f and g are o- and T -measurable, resp. Therefore,
1
(R) o T ,
which proves the claim.
Now that we knowF(x, y) = f(x)g(y) is (oT )-measurable, we can apply part (b) of the Fubini-Tonelli theorem
(A.13) to prove that F(x, y) = f(x)g(y) is integrable if one of the iterated integrals of [F(x, y)[ is nite. Indeed,
_
X
_
Y
[f(x)g(y)[ d(y) d(x) =
_
X
_
Y
[f(x)[[g(y)[ d(y) d(x)
=
_
X
[f(x)[
__
Y
[g(y)[ d(y)
_
d(x)
=
_
X
[f(x)[ d(x)
_
Y
[g(y)[ d(y) < ,
12
See also: November 97 (2), and others.
13
Im not sure if the claim is true unless the measure spaces are -nite, so Ill assume all measure spaces -nite.
In my opinion, the most useful version of the Fubini-Tonelli theorem is the one in Rudin [8], which assumes -nite measure spaces. There is a
version appearing in Royden [6] that does not require -niteness. Instead it begins with the assumption that f is integrable. To me, the theorem
in Rudin is much easier to apply. All you need is a function that is measurable with respect to the product -algebra S T , and from there, in a
single theorem, you get everything you need to answer any of the standard questions about integration with respect to a product measure.
8
1.1 1991 November 21 1 REAL ANALYSIS
which holds since f L
1
() and g L
1
(). The Fubini-Tonelli theorem then implies that F(x, y) L
1
( ).
Finally, we must prove that
_
F d() =
_
f d
_
g d. Since F(x, y) L
1
(), part (c) of the Fubini-Tonelli
theorem asserts that (x) =
_
Y
F(x, y) d(y) is dened almost everywhere, belongs to L
1
(), and, moreover,
_
XY
F d( ) =
_
X
_
Y
F(x, y) d(y) d(x).
Therefore,
_
XY
F d( ) =
_
X
_
Y
f(x)g(y) d(y) d(x)
=
_
X
f(x)
_
Y
g(y) d(y) d(x)
=
_
X
f(x) d(x)
_
Y
g(y) d(y).
.
9
1.2 1994 November 16 1 REAL ANALYSIS
1.2 1994 November 16
Masters students: Do any 5 problems.
Ph.D. students: Do any 6 problems.
1. Let E be a normed linear space. Show that E is complete if and only if, whenever

1
|x
n
| < , then

1
x
n
converges to an s E.
Solution: Suppose E is complete. Let x
n
E be absolutely convergent; i.e.,

|x
n
| < . We must

n=1
x
n
:= lim
N
N

n=1
x
n
= s E. (6)
Let S
N
=

N
n=1
x
n
. Then, for any j N,
|S
N+j
S
N
| =
_
_
_
_
_
N+j

n=N+1
x
n
_
_
_
_
_

N+j

n=N+1
|x
n
| 0
as N , since

|x
n
| < . Therefore, S
N
is a Cauchy sequence. Since E is complete, there is an s E
such that

n=1
x
n
= lim
N
S
N
= s.
Conversely, suppose whenever

1
|x
n
| < , then

1
x
n
converges to an s E. Let y
n
E be a Cauchy
sequence. That is, |y
n
y
m
| 0 as n, m . Let n
1
< n
2
< be a subsequence such that
n, m n
j
|y
n
y
m
| < 2
j
.
Next observe, for k > 1,
y
n
k
= y
n1
+ (y
n2
y
n1
) + (y
n3
y
n2
) + + (y
n
k
y
n
k1
) = y
n1
+
k1

j=1
(y
nj+1
y
nj
),
and

j=1
|y
nj+1
y
nj
| <

j=1
2
j
= 1
By hypothesis, this implies that
y
n
k
y
n1
=
k1

j=1
(y
nj+1
y
nj
) s E,
as k . We have thus found a subsequence y
n
k
y
n
having a limit in E. Finally, since y
n
is Cauchy,
it is quite easy to verify that y
n
must converge to the same limit. This proves that every Cauchy sequence in E
converges to a point in E. .
2. Let f
n
be a sequence of real continuous functions on a compact Hausdorff space X. Show that if f
1
f
2
f
3

, and f
n
(x) 0 for all x X, then f
n
0 uniformly.
10
1.2 1994 November 16 1 REAL ANALYSIS
3. Let f be integrable on the real line with respect to Lebesgue measure. Evaluate lim
n
_

f(x n)
_
x
1+|x|
_
dx.
Justify all steps.
Solution: Fix n > 0. Consider the change of variables, y = x n. Then dy = dx and x = y +n, so
_

f(x n)
x
1 +[x[
dx =
_

f(y)
y +n
1 +[y +n[
dy
=
_

n
f(y)
y +n
1 +y +n
dy +
_
n

f(y)
y +n
1 (y +n)
dy. (7)
Note that, when y n,
y+n
1+y+n
[0, 1), and increases to 1 as n tends to innity. Thus,
0 [f(y)[
y +n
1 +y +n
[f(y)[,
for all y n. Dene the function
14
g
n
(y) = f(y)
y +n
1 +y +n
1
[n,)
(y).
Then [g
n
[ [f[ and lim
n
g
n
= f. Therefore, by the dominated convergence theorem,
lim
n
_

n
f(y)
y +n
1 +y +n
dy = lim
n
_

g
n
(y) dy =
_

f(y) dy.
Next, consider the second term in (7). Dene the function
h
n
(y) = f(y)
y +n
1 (y +n)
1
(,n]
(y).
It is not hard to check that
[y +n[
[1 (y +n)[
1
(,n]
(y) [0, 1),
from which it follows that [h
n
[ [f[. Also, it is clear that, for all y,
lim
n
h
n
(y) = f(y) lim
n
y +n
1 (y +n)
1
(,n]
(y) = 0.
Therefore, the dominated convergence theorem implies that
lim
n
_
n

f(y)
y +n
1 (y +n)
dy = 0.
Combining the two results above, we see that lim
n
_

f(x n)
_
x
1+|x|
_
dx =
_

f(x) dx. .
Remark. Intuitively, this is the result we expect because the translation f(x n) = T
n
f(x) is merely shifting the
support of f to the right tail of the measure d :=
x
1+|x|
dx, and in the tail this measure looks like dx.
14
Here 1
A
(x) denotes the indicator function of the set A, which is 1 if x A and 0 if x / A.
11
1.3 1998 April 3 1 REAL ANALYSIS
1.3 1998 April 3
Instructions Do at least four problems in Part A, and at least two problems in Part B.
PART A
1. Let x
n

n=1
be a bounded sequence of real numbers, and for each positive n dene
x
n
= sup
kn
x
k
(a) Explain why the limit = lim
n
x
n
exists.
(b) Prove that, for any > 0 and positive integer N, there exists an integer k such that k N and [x
k
[ < .
2. Let C be a collection of subsets of the real line R, and dene
A

(C) =

A : C A and A is a -algebra of subsets of R.


(a) Prove that A

(C) is a -algebra, that C A

(C), and that A

(C) A for any other -algebra A containing


all the sets of C.
(b) Let O be the collection of all nite open intervals in R, and F the collection of all nite closed intervals in R.
Show that
A

(O) = A

(F).
3. Let (X, A, ) be a measure space, and suppose X =
n
X
n
, where X
n

n=1
is a pairwise disjoint collection of
measurable subsets of X. Use the monotone convergence theorem and linearity of the integral to prove that, if f is
a non-negative measurable real-valued function on X,
_
X
f d =

n
_
Xn
f d.
Solution:
15
Dene f
n
=

n
k=1
f
X
k
= f

n
1
X
k
. Then it is clear that the hypotheses of the monotone conver-
gence theorem are satised. That is, for all x X,
(i) 0 f
1
(x) f
2
(x) f(x), and
(ii) lim
n
f
n
(x) = f(x)
X
(x) = f(x).
15
Note that the hypotheses imply (E) =

E
f d is a measure (problem 4, Nov. 91), from which the desired conclusion immediately follows.
Of course, this does not answer the question as stated, since the examiners specically require the use of the MCT and linearity of the integral.
12
1.3 1998 April 3 1 REAL ANALYSIS
Therefore,

k=1
_
X
k
f d = lim
n
n

k=1
_
X
f
X
k
d
= lim
n
_
X
n

k=1
f
X
k
d (by linearity of the integral)
= lim
n
_
X
f
n
d (by denition of f
n
)
=
_
X
lim
n
f
n
d (by the monotone convergence theorem)
=
_
X
f d.
.
4. Using the Fubini/Tonelli theorems to justify all steps, evaluate the integral
_
1
0
_
1
y
x
3/2
cos
_
y
2x
_
dxdy.
Solution: By Tonellis theorem, if f(x, y) 0 is measurable and one of the iterated integrals
__
f(x, y) dxdy or
__
f(x, y) dy dx exists, then they both exist and are equal. Moreover, if one of the iterated integrals is nite, then
f(x, y) L
1
(dx, dy). Fubinis theorem states: if f(x, y) L
1
(dx, dy), then the iterated integrals exist and are
equal.
Now let g(x, y) = x
3/2
cos(y/2x), and apply the Tonelli theorem to the non-negative measurable function
[g(x, y)[ as follows:
_
1
0
_
x
0
[g(x, y)[ dy dx =
_
1
0
_
x
0
[x[
3/2

cos
_
y
2x
_

dy dx
_
1
0
_
x
0
x
3/2
1 dy dx =
_
1
0
x
1/2
dx = 2.
Thus one of the iterated integrals of [g(x, y)[ is nite which, by the Tonelli theorem, implies g(x, y) L
1
(dx, dy).
Therefore, the Fubini theorem applies to g(x, y), and gives the rst of the following equalities:
_
1
0
_
1
y
x
3/2
cos
_
y
2x
_
dxdy =
_
1
0
_
x
0
x
3/2
cos
_
y
2x
_
dy dx
=
_
1
0
x
3/2

2x

_
sin
_
y
2x
__
y=x
y=0
dx
=
_
1
0
2

x
1/2
dx
=
2

_
2x
1/2
_
x=1
x=0
=
4

.
.
13
1.3 1998 April 3 1 REAL ANALYSIS
5. Let I be the interval [0, 1], and let C(I), C(I I) denote the spaces of real valued continuous functions on I and
I I, respectively, with the usual supremum norm on these spaces. Show that the collection of nite sums of the
form
f(x, y) =

i
(x)
i
(y),
where
i
,
i
C(I) for each i, is dense in C(I I).
6. Let m be Lebesgue measure on the real line R, and for each Lebesgue measurable subset E of R dene
(E) =
_
E
1
1 +x
2
dm(x).
Show that m is absolutely continuous with respect to , and compute the Radon-Nikodym derivative dm/d.
Solution: Obviously both measures are non-negative. We must rst prove m . To this end, suppose m(E) >
0, where E M, the -algebra of Lebesgue measurable sets. Then, if we can show (E) > 0, this will establish
that the implication (E) = 0 m(E) = 0 holds for all E M; i.e., m .
For n = 1, 2, . . ., dene
A
n
=
_
x R :
1
n + 1
<
1
1 +x
2

1
n
_
.
Then A
i
A
j
= for all i ,= j in N, and, for all n = 1, 2, . . .,
(A
n
)
1
n + 1
m(A
n
).
Also, R = A
n
, since 0 <
1
1+x
2
1 holds for all x R. Therefore,
(E) = (E (
n
A
n
)) = (
n
(A
n
E)) =

n
(A
n
E).
The last equality might need a bit of justication: Since f(x) =
1
1+x
2
is continuous, hence measurable, the sets
A
n
are measurable. Therefore, the last equality holds by countable additivity of disjoint measurable sets.
Now note that m(E) =

m(A
n
E) > 0 implies the existence of an n Nsuch that m(A
n
E) > 0. Therefore,
(E) (A
n
E)
1
n + 1
m(A
n
E) > 0,
which proves that m . By the Radon-Nikodym theorem (A.1), there is a unique h L
1
() such that
m(E) =
_
hd, and
_
f dm =
_
fhd f L
1
(m).
In particular, if E Mand f(x) =
1
1+x
2

E
, then
(E) =
_
E
1
1 +x
2
dm(x) =
_
E
h(x)
1 +x
2
d(x).
That is,
_
E
d =
_
E
h(x)
1+x
2
d(x) holds for all measurable sets E, which implies
16
that,
h(x)
1+x
2
= 1 holds for
-almost every x R. Therefore,
dm
d
(x) = h(x) = 1 +x
2
.
One nal note: h is uniquely dened only up to an equivalence class of functions that are equal to 1+x
2
, -a.e. .
16
Recall the standard result: if f and g are integrable functions such that

E
f =

E
g holds for all measurable sets E, then f = g, -a.e.
This is an exam problem, but I cant remember on which exam it appears. When I come across it again Ill put a cross reference here.
14
1.3 1998 April 3 1 REAL ANALYSIS
PART B
7. Let (x, y) = x
2
y be dened on the square S = [0, 1] [0, 1] in the plane, and let mbe two-dimensional Lebesgue
measure on S. Given a Borel subset E of the real line R, dene
(E) = m(
1
(E)).
(a) Show that is a Borel measure on R.
(b) Let
E
denote the characteristic function of the set E. Show that
_
R

E
d =
_
S

E
dm.
(c) Evaluate the integral
_

t
2
d(t).
8. Let f be a real valued and increasing function on the real line R, such that f() = 0 and f() = 1. Prove that
f is absolutely continuous on every closed nite interval if and only if
_
R
f

dm = 1.
Solution:
17
First note that f is increasing, so f

exists for a.e. x R, and f

(x) 0 wherever f

exists. Also, f

is measurable. To see this, dene


g(x) = limsup
n
[f(x + 1/n) f(x)] n.
As a lim sup of a sequence of measurable functions, g is measurable (Rudin [8], theorem 1.14?). Let E be the set
on which f

exists. Then m(R E) = 0, and f

= g on E (by the denition of derivative), so f

is measurable.
() Suppose
_
R
f

dm = 1. We must show f AC[a, b] for all < a < b < . First, check that
f

L
1
(R), since
1 =
_
R
f

dm =
_
R\E
f

dm+
_
E
f

dm =
_
E
f

dm,
and, since f is increasing, f

0 on E, so
_
R
[f

[dm =
_
R\E
[f

[dm+
_
E
[f

[dm =
_
E
[f

[dm =
_
E
f

dm = 1.
Thus, f

L
1
(R) as claimed. A couple of lemmas will be needed to complete the direction of the proof. The
rst is proved in the appendix (sec. A), while the second can be found in Royden [6] on page 100.
Lemma 1.2 Let f : R R be a function. If f is differentiable on [a, b], f

L
1
([a, b]), and
_
x
a
f

(t)dt =
f(x) f(a) for a x b, then f AC[a, b].
17
I have worked this problem a number of times, and what follows is the clearest and most instructive proof Ive come up with. Its by no means
the shortest, most elegant solution, and probably not the type of detailed answer one should give on an actual exam. However, some of the facts that
I prove in detail have appeared as separate questions on other exams, so the proofs are worth knowing.
15
1.3 1998 April 3 1 REAL ANALYSIS
The converse of this lemma is also true.
18
Lemma 1.3 If f : R R is increasing and f

L
1
([a, b]), then
_
x
a
f

(t)dt f(x) f(a).


To nish the direction of the proof, by lemma 1.2, it sufces to show that
_
R
f

dm = 1 implies
_
b
a
f

(t)dt =
f(b) f(a) holds for all < a < b < . By lemma 1.3, we have
_
b
a
f

(t)dt f(b) f(a), so we need only


show that strict inequality cannot hold. Suppose, by way of contradiction, that
_
b
a
f

(t)dt < f(b) f(a) holds for


some < a < b < . Then,
1 =
_
R
f

dm =
_
a

dm+
_
b
a
f

dm+
_

b
f

dm
< [f(a) f()] + [f(b) f(a)] + [f() f(b)]
= f() f() = 1.
This contradiction proves that
_
R
f

dm = 1 implies
_
b
a
f

(t)dt = f(b) f(a) holds for all < a < b < , as


desired.
() Now assume f AC[a, b] for all < a < b < . We must show
_
R
f

dm = 1. By assumption
f() f() = 1, so this is equivalent to showing
lim
x
_
x
x
f

(t)dm(t) = lim
x
[f(x) f(x)].
Let x R, x > 0, and f AC[x, x]. Then we claim f(x) f(x) =
_
x
x
f

dm. Assuming the claim is true


(see Royden [6], p. 110 for the proof), we have
1 = lim
x
[f(x) f(x)] = lim
x
_
x
x
f

(t)dm(t) =
_
R
f

dm.
.
9. Let F be a continuous linear functional on the space L
1
[1, 1], with the property that F(f) = 0 for all odd
functions f in L
1
[1, 1]. Show that there exists an even function such that
F(f) =
_
1
1
f(x)(x) dx, for all f L
1
[1, 1].
[Hint: One possible approach is to use the fact that any function in L
p
[1, 1] is the sum of an odd function and an
even function.]
Solution: Since F L
1
[1, 1]

, then by the Riesz representation theorem


19
there is a unique h L

[1, 1]
such that
F(f) =
_
1
1
f(x)h(x) dx (f L
1
[1, 1])
18
See Folland [4] for a nice, concise treatment of the fundamental theorem of calculus for Lebesgue integration.
19
See problem 3 of section 1.5.
16
1.3 1998 April 3 1 REAL ANALYSIS
Now (using the hint) write h = +, where and are the even and odd functions
(x) =
h(x) +h(x)
2
and (x) =
h(x) h(x)
2
.
Similarly, let f = f
e
+ f
o
be the decomposition of f into a sum of even and odd functions. Then, by linearity of
F, and since F(f
o
) = 0 by hypothesis,
F(f) = F(f
e
) +F(f
o
) = F(f
e
) =
_
1
1
f
e
h =
_
1
1
f
e
+
_
1
1
f
e
.
Now note that f
e
is an odd function (since its an even times an odd), so
_
1
1
f
e
= 0, since [1, 1] is symmetric.
Similarly,
_
1
1
f
o
= 0. Therefore,
F(f) = F(f
e
) =
_
1
1
f
e
=
_
1
1
f
e
+
_
1
1
f
o
=
_
1
1
(f
e
+f
o
) =
_
1
1
f.
.
17
1.4 2000 November 17 1 REAL ANALYSIS
1.4 2000 November 17
Do as many problems as you can. Complete solutions to ve problems would be considered a good performance.
1. (a)
20
State the inverse function theorem.
(b) Suppose L : R
3
R
3
is an invertible linear map and that g : R
3
R
3
has continuous rst order partial
derivatives and satises |g(x)| C|x|
2
for some constant C and all x R
3
. Here |x| denotes the usual
Euclidean norm on R
3
. Prove that f(x) = L(x) +g(x) is locally invertible near 0.
Solution:
(a) (Inverse function theorem (IFT) of calculus)
21
Let f : E R
n
be a C
1
-mapping of an open set E R
n
. Suppose that f

(a) is invertible for some a E and


that f(a) = b. Then,
(i) there exist open sets U and V in R
n
such that a U, b V , and f maps U bijectively onto V , and
(ii) if g is the inverse of f (which exists by (i)), dened on V by g(f(x)) = x, for x U, then g C
1
(V ).
(b) First note that L and g both have continuous rst order partial derivatives; i.e., L, g C
1
(R
3
). Therefore, the
derivative of f = L +g,
f

(x) J
f
(x)
_
f
i
x
j
_
3
i,j=1
exists. Furthermore, J
f
(x) is continuous in a neighborhood of the zero vector, because this is true of the partials of
g(x), and the partials of L(x) are the constant matrix L. Therefore, f C
1
(R
3
). By the IFT, then, we need only
show that f

(0) is invertible. Since f

(x) = L + g

(x), we must show f

(0) = L + g

(0) is invertible. Consider


the matrix g

(0) = J
g
(0). We claim, J
g
(0) = 0. Indeed, if x
1
, x
2
, x
3
are the elementary unit vectors (also known
as i, j, k), then the elements of J
g
(0) are
g
i
x
j
(0) = lim
h0
g
i
(0 +hx
j
) g
i
(0)
h
= lim
h0
g
i
(hx
j
)
h
. (8)
The second equality follows by the hypothesis that g is continuous and satises |g(x)| C|x|
2
, which implies
that g(0) = 0. Finally, to show that (8) is zero, consider
[g
i
(hx
j
)[ |g(hx
j
)| C|hx
j
|
2
= C[h[
2
,
which implies
[g
i
(hx
j
)[
[h[
C
[hx
j
[
2
[h[
= C[h[ 0, as h 0.
This proves that f

(0) = L, which is invertible by assumption, so the IFT implies that f(x) is locally invertible
near 0. .
2. Let f be a differentiable real valued function on the interval (0, 1), and suppose the derivative of f is bounded on
this interval. Prove the existence of the limit L = lim
x0
+ f(x).
20
The inverse function theorem does not appear on the syllabus and, as far as I know, this is the only exam problem in which it has appeared. The
implicit function theorem does appear on the syllabus, but I have never encountered an exam problem that required it.
21
See Rudin [7].
18
1.4 2000 November 17 1 REAL ANALYSIS
3. Let f and g be Lebesgue integrable functions on [0, 1], and let F and G be the integrals
F(x) =
_
x
0
f(t) dt, G(x) =
_
x
0
g(t) dt.
Use Fubinis and/or Tonellis theorem to prove that
_
1
0
F(x)g(x) dx = F(1)G(1)
_
1
0
f(x)G(x) dx.
Other approaches to this problem are possible, but credit will be given only to solutions based on these theorems.
4. Let (X, A, ) be a nite measure space and suppose is a nite measure on (X, A) that is absolutely continuous
with respect to . Prove that the norm of the Radon-Nikodym derivative f =
_
d
d
_
is the same in L

() as it is in
L

().
5. Suppose that f
n
is a sequence of Lebesgue measurable functions on [0, 1] such that lim
n
_
1
0
[f
n
[ dx = 0 and
there is an integrable function g on [0, 1] such that [f
n
[
2
g, for each n. Prove that lim
n
_
1
0
[f
n
[
2
dx = 0.
6. Denote by T
e
the family of all even polynomials. Thus a polynomial p belongs to T
e
if and only if p(x) =
p(x)+p(x)
2
for all x. Determine, with proof, the closure of T
e
in L
1
[1, 1]. You may use without proof the fact
that continuous functions on [1, 1] are dense in L
1
[1, 1].
7. Suppose that f is real valued and integrable with respect to Lebesgue measure m on R and that there are real
numbers a < b such that
a m(U)
_
U
f dm b m(U),
for all open sets U in R. Prove that a f(x) b a.e.
19
1.5 2001 November 26 1 REAL ANALYSIS
1.5 2001 November 26
Instructions Masters students do any 4 problems Ph.D. students do any 5 problems. Use a separate sheet of paper for
each new problem.
1. Let f
n
be a sequence of Lebesgue measurable functions on a set E R, where E is of nite Lebesgue mea-
sure. Suppose that there is M > 0 such that [f
n
(x)[ M for n 1 and for all x E, and suppose that
lim
n
f
n
(x) = f(x) for each x E. Use Egoroff s theorem to prove that
_
E
f(x) dx = lim
n
_
E
f
n
(x) dx.
Solution: First note that [f(x)[ M for all x E. To see this, suppose its false for some x
0
E, so that
[f(x
0
)[ > M. Then there is some > 0 such that [f(x
0
)[ = M +. By the triangle inequality, then, for all n N,
[f(x
0
) f
n
(x
0
)[ [[f(x
0
)[ [f
n
(x
0
)[[ = [M + [f
n
(x
0
)[[ ,
which contradicts f
n
(x
0
) f(x
0
). Thus, [f(x)[ M for all x E.
Next, x > 0. By Egoroffs theorem (A.8), there is a G E such that (E G) < and f
n
f uniformly
on G. Furthermore, since [f
n
[ M and [f[ M and (E) < , its clear that f
n
L
1
and f L
1
, so the
following inequalities make sense (here were using the notation |f|
G
= sup[f(x)[ : x G):

_
E
f d
_
E
f
n
d

_
E
[f f
n
[ d =
_
E\G
[f f
n
[ d +
_
G
[f f
n
[ d

_
E\G
[f[ d +
_
E\G
[f
n
[ d +|f(x) f
n
(x)|
G
(G)
2M(E G) +|f(x) f
n
(x)|
G
(G)
< +|f(x) f
n
(x)|
G
(G).
Finally, (G) (E) < and |f(x) f
n
(x)|
G
0, which proves that
_
E
f
n
d
_
E
f d. .
2. Let f(x) be a real-valued Lebesgue integrable function on [0, 1].
(a) Prove that if f > 0 on a set F [0, 1] of positive measure, then
_
F
f(x) dx > 0.
(b) Prove that if
_
x
0
f(x) dx = 0, for each x [0, 1],
then f(x) = 0 for almost all x [0, 1].
Solution: (a) Dene F
n
= x F : f(x) > 1/n. Then
F
1
F
2

_
n
F
n
= F,
and m(F) > 0 implies
0 < m(F) = m(
n
F
n
)

n
m(F
n
).
20
1.5 2001 November 26 1 REAL ANALYSIS
Therefore, m(F
k
) > 0 for some k N, and then it follows from the denition of F
k
that
0 <
1
k
m(F
k
)
_
F
k
f dm
_
F
f dm.
.
(b) Suppose there is a subset E [0, 1] of positive measure such that f > 0 on E. Then part (a) implies
_
E
f dm > 0. Let F E be a closed subset of positive measure. (That such a closed subset exists follows from
Prop. 3.15 of Royden [6].) Then, again by (a),
_
F
f dm > 0. Now consider the set G = [0, 1] F, which is open
in [0, 1], and hence
22
is a countable union of disjoint open intervals; i.e., G =
n
(a
n
, b
n
). Therefore,
0 =
_
[0,1]
f dm =

n
_
(an,bn)
f dm+
_
F
f dm,
so
_
F
f dm > 0 implies

n
_
(an,bn)
f dm < 0.
Thus,
_
(a
k
,b
k
)
f dm < 0 for some (a
k
, b
k
) [0, 1]. On the other hand,
_
(a
k
,b
k
)
f dm =
_
b
k
0
f(x) dm(x)
_
a
k
0
f(x) dm(x).
By the initial hypothesis, both terms on the right are zero, which gives the desired contradiction. .
3. State each of the following:
(a) The Stone-Weierstrass theorem
(b) The Lebesgue (dominated) convergence theorem
(c) H olders inequality
(d) The Riesz representation theorem for L
p
(e) The Hahn-Banach theorem.
Solution:
23
(a) (Stone-Weierstrass theorem)
Let X be a compact Hausdorff space and let / be a closed subalgebra of functions in C(X, R) which separates
points. Then either / = C(X, R), or / = f C(X, R) : f(x
0
) = 0 for some x
0
X. The rst case occurs iff
/ contains the constant functions.
(b) (Lebesgue dominated convergence theorem)
24
Let f
n
be a sequence of measurable functions on (X, M, )
such that f
n
f a.e.. If there exists g L
1
(X, M, ) such that [f
n
(x)[ g(x) holds for all x X and
n = 1, 2, . . .. Then f
n
L
1
, f L
1
, lim
_
X
f
n
d =
_
X
f d, and |f
n
f|
1
0.
(c) (H olders inequality)
Let f and g be measurable functions.
22
Every open set of real numbers is the union of a countable collection of disjoint open intervals (Royden [6], Prop. 8, page 42).
23
The presentations of (a) and (c) in Folland [4] are especially nice. For (b) and (e), as well as (c), I like Rudin [8]. A version of (d) appears in
Royden [6].
24
See theorem A.7 for a more general version.
21
1.5 2001 November 26 1 REAL ANALYSIS
(i) If 1 < p < and
1
p
+
1
q
= 1, then |fg|
1
= |f|
p
|g|
q
. Thus, if f L
p
and g L
q
, then fg L
1
(ii) If p = and if f L

and g L
1
, then [fg[ |f|

[g[, so |fg|
1
|f|

|g|
1
.
(d) (Riesz representation theorem for L
p
)
25
Suppose 1 < p < and
1
p
+
1
q
= 1. If is a linear functional on L
p
, then there is a unique g L
q
such that
f =
_
fg d (f L
p
).
(e) (Hahn-Banach theorem)
Suppose X is a normed linear space, Y X is a subspace, and T : Y R is a bounded linear functional.
Then there exists a bounded linear functional

T : X R such that

T(y) = T(y) for all y Y , and such that
|

T|
X
= |T|
Y
, where |

T|
X
and |T|
Y
are the usual operator norms,
|

T|
X
= sup[

Tx[ : x X, |x| 1 and |T|


Y
= sup[Tx[ : x Y, |x| 1.
4. (a) State the Baire category theorem.
(b) Prove the following special case of the uniform boundedness theorem: Let X be a (nonempty) complete metric
space and let F C(X). Suppose that for each x X there is a nonnegative constant M
x
such that
[f(x)[ M
x
for all f F.
Prove that there is a nonempty open set G X and a constant M > 0 such that
[f(x)[ M holds for all x G and for all f F.
Solution:
26
(a) (Baire category theorem)
If X is a complete metric space and A
n
is a collection of open dense subsets, then

n=1
A
n
is dense in X.
Corollary 1. If X is a complete metric space and G X is a non-empty open subset and G =

n=1
G
n
then

G
n
o
,= for at least one n N.
Corollary 2. A nonempty complete metric space is not a countable union of nowhere dense sets.
(b) Dene A
m
= x X : [f(x)[ m, f F. Then X =

m=1
A
m
, since for every x there is a nite
number M
x
such that [f(x)[ M
x
for all f F. Now note that A
m
=

fF
x X : [f(x)[ m, and, since f
and a [a[ are continuous functions, each x X : [f(x)[ m is closed, so A
m
is closed. Therefore, corollary
2 of the Baire category theorem implies that there must be some m N such that A

m
,= , so the set G = A

m
and
the number M = m satisfy the given criteria. .
25
Note to self: add case p =
26
See Royden [6], 7.8, for an excellent treatment of this topic. Part (b) of this problem appears there as theorem 32, and another popular exam
question is part c of problem 37.
22
1.5 2001 November 26 1 REAL ANALYSIS
5. Prove or disprove:
(a) L
2
convergence implies pointwise convergence.
(b)
lim
n
_

0
sin(x
n
)
x
n
dx = 0.
(c) Let f
n
be a sequence of measurable functions dened on [0, ). If f
n
0 uniformly on [0, ), as n ,
then
lim
_
[0,)
f
n
(x) dx =
_
[0,)
limf
n
(x) dx.
Solution:
(a) This is false, as the following example demonstrates: For each k N, dene f
k,j
=
[
j1
k
,
j
k
)
for j = 1, . . . , k,
and let g
n
be the sequence dened by
g
1
= f
1,1
,
g
2
= f
2,1
, g
3
= f
2,2
,
g
4
= f
3,1
, g
5
= f
3,2
, g
6
= f
3,3
,
g
7
= f
4,1
, . . .
Then
_
[f
k,j
[
2
d = 1/k for each j = 1, . . . , k, so |f
k,j
|
2
= 1/

k 0, as k . Therefore |g
n
|
2
0 as
n . However, g
n
does not converge pointwise since, for every x [0, 1] and every N N, we can always
nd some k N and j 1, . . . , k such that g
n
(x) = f
k,j
(x) = 1 with n N, and we can also nd a k

N
and j

1, . . . , k

such that g
n
(x) = f
k

,j
(x) = 0 with n

N. .
(b) For any xed 0 < x < 1, lim
n
x
n
= 0. Also,
sin t
t
1, as t 0, which can be proved by LHopitals rule.
Together, these two facts yield
lim
n
sin x
n
x
n
= 1.
Now, recall that [ sin [ [[ for all real . Indeed, since sin =
_

0
cos xdx, we have, for 0,
[ sin [
_

0
[ cos x[ dx
_

0
1 dx = ,
and, for < 0,
[ sin [ = [ sin()[ [ [ = [[.
In particular, for any 0 < x < 1,
| sin x
n
|
|x
n
|
1. Therefore, we can apply the dominated convergence theorem to the
function
sin x
n
x
n
, to obtain
lim
n
_
1
0
sin x
n
x
n
dx =
_
1
0
1 dx = 1. (9)
Next consider the part of the integral over 1 x < N, for any real N > 1. Fix n 2. The change of variables
u = x
n
results in du = nx
n1
dx, and, since u
1/n
= x, we have x
n1
= u
1
1
n
. Therefore,
_
N
1
sin x
n
x
n
dx =
_
N
n
1
sin u
u
du
nu
1
1
n
=
1
n
_
N
n
1
sin u
u
2
1
n
du.
23
1.5 2001 November 26 1 REAL ANALYSIS
Now,
lim
N
1
n

_
N
n
1
sin u
u
2
1
n
du

lim
N
1
n
_
N
n
1
u
1
n
2
du = lim
N
1
n
u
1
n
1
1
n
1

N
n
1
=
1
n 1
.
Therefore,

_

1
sin x
n
x
n
dx

1
n 1
,
and so,
lim
n
_

1
sin x
n
x
n
dx = 0. (10)
Combining results (9) and (10) yields
lim
n
_

0
sin x
n
x
n
dx = 1.
.
(c) This is false, as the following example demonstrates: Let f
n
=
1
n

[0,n)
. Then f
n
0 uniformly and so
_
limf
n
= 0. On the other hand,
_
f
n
= 1 for all n N. Therefore, lim
_
f
n
= 1 ,= 0 =
_
limf
n
. .
6. Let f : H H be a bounded linear functional on a separable Hilbert space H (with inner product denoted by
, ). Prove that there is a unique element y H such that
f(x) = x, y for all x H and |f| = |y|.
Hint. You may use the following facts: A separable Hilbert space, H, contains a complete orthonormal sequence,

k=1
, satisfying the following properties: (1) If x, y H and if x,
k
= y,
k
for all k, then x = y. (2)
Parsevals equality holds; that is, for all x H, x, x =

k=1
a
2
k
, where a
k
= x,
k
.
Solution: Dene y =

k=1
f(
k
)
k
, and check that this y H has the desired properties.
First observe that, by properties (1) and (2) given the hint, any x H can be written as x =

k=1
a
k

k
, where
a
k
= x,
k
, for each k N. Therefore, by linearity of f,
f(x) = f(

k
a
k

k
) =

k
a
k
f(
k
). (11)
Now
x, y =

k
a
k

k
, y =

k
a
k

k
, y, (12)
and, by denition of y,

k
, y =
k
,

j
f(
j
)
j
=

j
f(
j
)
k
,
j
= f(
k
). (13)
The last equality holds by orthonormality; i.e.,
k
,
j
is 1 when j = k and 0 otherwise. Putting it all together,
we see that, for every x H,
f(x) =

k
a
k
f(
k
) (11)
=

k
a
k

k
, y (13)
= x, y (12)
24
1.5 2001 November 26 1 REAL ANALYSIS
Moreover, this y is unique. For, suppose there is another y

H such that f(x) = x, y

for all x X. Then


x, y = f(x) = x, y

for all x X. In particular,


k
, y = f(
k
) =
k
, y

for each k N, which, by


property (1) of the hint, proves that y = y

.
Finally, we must show |f| = |y|. Observe,
|f| = sup
xX
[f(x)[ : |x| 1 = sup
xX
[f(x)[
|x|
= sup
xX
[(x, y)[
|x|
,
and recall that [(x, y)[ |x||y| holds for all x, y X. Whence,
|f| = sup
xX
[(x, y)[
|x|
|y|. (14)
On the other hand,
|f| = sup
xX
[(x, y)[
|x|

[(y, y)[
|y|
=
|y|
2
|y|
= |y|. (15)
Together, (14) and (15) give |f| = |y|, as desired. .
7. Let X be a normed linear space and let Y be a Banach space. Let
B(X, Y ) = A[ A : X Y is a bounded linear operator.
Then with the norm |A| = sup
x1
|Ax|, B(X, Y ) is a normed linear space (you need not show this). Prove
that B(X, Y ) is a Banach space; that is, prove that B(X, Y ) is complete.
Solution: Let T
n
B(X, Y ) be a Cauchy sequence; i.e., |T
n
T
m
| 0 as m, n . Fix x X. Then,
|T
n
x T
m
x|
Y
|T
n
T
m
||x|
X
0, as n, m .
Therefore, the sequence T
n
x Y is a Cauchy sequence in (Y, | |
Y
). Since the latter is complete, the limit
lim
n
T
n
x = y Y exists. Dene T : X Y by Tx = lim
n
T
n
x, for each x X. To complete the proof,
we must check that T is linear, bounded, and satises lim
n
|T
n
T| = 0.
T is linear:
For x
1
, x
2
X,
T(x
1
+x
2
) = lim
n
T
n
(x
1
+x
2
)
= lim
n
(T
n
x
1
+T
n
x
2
) ( T
n
is linear)
= lim
n
T
n
x
1
+ lim
n
T
n
x
2
( both limits exist)
= Tx
1
+Tx
2
.
T is bounded:
First, note that |T
n
| is a Cauchy sequence of real numbers, since [ |T
n
| |T
m
| [ |T
n
T
m
| 0,
as n, m . Therefore, there is a c R such that |T
n
| c, as n . For some N N, then,
|T
n
| c + 1 for all n N. Thus,
|T
n
x|
Y
|T
n
||x|
X
(c + 1)|x|
X
(x X). (16)
25
1.5 2001 November 26 1 REAL ANALYSIS
Now, by denition, T
n
x Tx, for all x X and, since the norm | |
Y
is uniformly continuous,
27
|T
n
x|
Y
|Tx|
Y
(x X). (17)
Taken together, (16) and (17) imply |Tx|
Y
(c + 1)|x|
X
, for all x X. Therefore, T is bounded.
lim
n
|T
n
T| = 0:
Fix > 0 and choose N N such that n, m N implies |T
n
T
m
| < . Then,
|T
n
x T
m
x| |T
n
T
m
||x|
X
< |x|
X
holds for all n, m N, and x X. Letting m go to innity, then,
|T
n
x Tx| = lim
m
|T
n
x T
m
x| |x|
X
.
That is, |T
n
x Tx| |x|
X
, for all n N and x X. Whence, |T
n
T| for all n N.
.
27
Proof: | a
Y
b
Y
| a b
Y
(a, b Y ).
26
1.6 2004 April 19 1 REAL ANALYSIS
1.6 2004 April 19
Instructions. Use a separate sheet of paper for each new problem. Do as many problems as you can. Complete
solutions to ve problems will be considered as an excellent performance. Be advised that a few complete and well
written solutions will count more than several partial solutions.
Notation: f C(X) means that f is a real-valued, continuous function dened on X.
1. (a) Let S be a (Lebesgue) measurable subset of R and let f, g : S R be measurable functions. Prove that (i)
f +g is measurable and (ii) if C(R), then (f) is measurable.
(b) Let f : [a, b] [, ] be a measurable function. Suppose that f takes the value only on a set of
(Lebesgue) measure zero. Prove that for any > 0 there is a positive number M such that [f[ M, except on a
set of measure less than .
Solution:
(a) Proof 1: Since f and g are real measurable functions of S, and since the mapping : R R R dened by
(x, y) = x+y is continuous, theorem A.3 implies that the function f +g = (f, g) is measurable. If C(R),
then (f) is measurable by part (b) of theorem A.2.
Proof 2: Let q
i

i=1
be an enumeration of the rationals. Then, for any R,
x S : f(x) +g(x) < =

_
i=1
x S : f(x) < q
i
x S : g(x) < q
i
.
Since each set on the right is measurable, and since -algebras are closed under countable unions and intersections,
x S : f(x) +g(x) < is measurable. Since was arbitrary, f +g is measurable.
The function f is measurable if and only if, for any open subset U of R, the set (f)
1
(U) is measurable. Let U
be open in R. Then
1
(U) is open, since C(R), and so (f)
1
(U) = f
1
(
1
(U)) is measurable, since f
is measurable. Therefore, f is measurable.
(b) Fix > 0. For n N, dene A
n
= x [a, b] : [f(x)[ n. Then
[a, b] =

_
n=1
A
n
A

, (18)
where
28
A

= x : f(x) = . Also, A
1
A
2
and, since f is measurable, each A
n
is measurable.
Therefore, (A
n
) (
n
A
n
), as n . Note that all sets are contained in [a, b] and thus have nite measure.
Let M N be such that (
n
A
n
) (A
M
) < . Then [f[ M except on [a, b] A
M
, and by (18),
([a, b] A
M
) = (
n
A
n
A

A
M
)
(
n
A
n
A
M
) +(A

)
= (
n
A
n
A
M
) < .
The second equality holds since we assumed f(x) = only on a set of measure zero; i.e., (A

) = 0. .
28
Since f is an extended real valued function, we must not forget to include A, without which the union in (18) would not be all of [a, b].
27
1.6 2004 April 19 1 REAL ANALYSIS
2. (a) State Egorovs theorem.
(b) State Fatous lemma.
(c) Let f
n
L
p
[0, 1], where 1 p < . Suppose that f
n
f a.e., where f L
p
[0, 1]. Prove that
|f
n
f|
p
0 if and only if |f
n
|
p
|f|
p
.
Solution:
(a) See theorem A.8.
(b) See theorem A.6.
(c) () By the Minkowsky inequality, |f
n
|
p
= |f
n
f + f|
p
|f
n
f|
p
+ |f|
p
. Similarly, |f|
p

|f
n
f|
p
+|f
n
|
p
. Together, the two inequalities yield [|f
n
|
p
|f|
p
[ |f
n
f|
p
. Therefore, |f
n
f|
p
0
implies [|f
n
|
p
|f|
p
[ 0. This proves necessity.
() I know of three proofs of sufciency. The second is similar to the rst, only much shorter as it exploits the full
power of the general version of Lebesgues dominated convergence theorem, whereas the rst proof merely relies
on Fatous lemma.
29
The third proof uses both Fatous lemma and Egoroffs theorem, so, judging from parts (a)
and (b), this may be closer to what the examiners had in mind. Note that none of the proofs use the assumption that
the measure space is nite, so we may as well work in the more general space L
p
(X, M, ).
Both proofs 1 and 2 make use of the following:
Lemma 1.4 If , [0, ) and 1 p < , then ( +)
p
2
p1
(
p
+
p
).
Proof: When p 1, (x) = x
p
is convex on [0, ). Thus, for all , [0, ),

+
2

p
=

+
2

1
2
[() +()] =
1
2
(
p
+
p
).
When , R, the triangle inequality followed by the lemma yields
[ [
p
[[[ +[[[
p
2
p1
([[
p
+[[
p
). (19)
Proof 1: By (19),
[f
n
f[
p
2
p1
([f
n
[
p
+[f[
p
).
In particular, f
n
f L
p
, for each n N. Moreover, the functions
g
n
= 2
p1
([f
n
[
p
+[f[
p
) [f
n
f[
p
. (20)
are non-negative. Now notice that limg
n
= 2
p
[f[
p
. Applying Fatous lemma to (20), then,
_
2
p
[f[
p
=
_
limg
n
lim
_
g
n
= lim
_
_
2
p1
([f
n
[
p
+[f[
p
) [f
n
f[
p
_
.
Since |f
n
|
p
|f|
p
, this implies
2
p
_
[f[
p
2
p
_
[f[
p
lim
_
[f
n
f[
p
.
Equivalently 0 lim
_
[f
n
f[
p
. This proves |f
n
f|
p
0. .
29
Disclaimer: I made up the rst proof, so you should check it carefully for yourself and decide whether you believe me.
28
1.6 2004 April 19 1 REAL ANALYSIS
Proof 2: By (19),
[f
n
f[
p
2
p1
([f
n
[
p
+[f[
p
). (21)
In particular, f
n
f L
p
, for each n N. Dene the functions
g
n
= 2
p1
([f
n
[
p
+[f[
p
) and g = 2
p
[f[
p
.
Then g
n
g a.e., and |f
n
|
p
|f|
p
implies
_
g
n

_
g. Also, g
n
[f
n
f[
p
0 a.e., by (21). Therefore,
the dominated convergence theorem (theorem A.7) implies
_
[f
n
f[
p
0. .
Proof 3: Since f L
p
, for all > 0, there is a number > 0 and a set B Mof nite measure such that f is
bounded on B,
_
X\B
[f[
p
d < /2, and
_
E
[f[
p
d < /2, for all E Mwith E < . By Egoroffs theorem,
there is a set A B such that (B A) < and f
n
f uniformly on A. Therefore,
_
X
[f[
p
=
_
X\B
[f[
p
+
_
B\A
[f[
p
+
_
A
[f[
p
< /2 +/2 +
_
A
[f[
p
+ lim
_
A
[f
n
[
p
, (22)
since, by Fatous lemma,
_
A
[f[
p
=
_
A
lim[f
n
[
p
lim
_
A
[f
n
[
p
. By hypothesis, |f
n
|
p
|f|
p
. Therefore
lim
_
A
[f
n
[
p
= lim
_
_
X
[f
n
[
p

_
X\A
[f
n
[
p
_
=
_
X
[f[
p
lim
_
X\A
[f
n
[
p
.
By (22), then,
_
X
[f[
p
< +
_
X
[f[
p
lim
_
X\A
[f
n
[
p
.
Therefore, lim
_
X\A
[f
n
[
p
< (since f L
p
). Finally, note that
|f
n
f|
p
= |(f
n
f)
A
+ (f
n
f)
X\A
|
p
|(f
n
f)
A
|
p
+|(f
n
f)
X\A
|
p
(Minkowsky)
|(f
n
f)
A
|
p
+|f
n

X\A
|
p
+|f
X\A
|
p
.
Therefore,
lim|f
n
f|
p
lim[f
n
(x) f(x) : x A(A)
1/p
+ lim
_
_
X\A
[f
n
[
p
_
1/p
+
_
_
X\A
[f[
p
_
1/p
.
The rst term on the right goes to zero since f
n
f uniformly on A. The other terms are bounded by 2
1/p
. .
29
1.6 2004 April 19 1 REAL ANALYSIS
3. (a) Let S = [0, 1] and let f
n
L
p
(S), where 1 < p < . Suppose that f
n
f a.e. on S, where f L
p
(S). If
there is a constant M such that |f
n
|
p
M for all n, prove that for each g L
q
(S),
1
p
+
1
q
= 1, we have
lim
n
_
S
f
n
g =
_
S
fg.
(b) Show by means of an example that this result is false for p = 1.
Solution:
(a) Since g L
q
(S), for all > 0 there exists > 0 such that if A is a measurable set with A < then
_
A
[g[
q
d < .
Let B
0
S denote the set on which f
n
does not converge to f. Let B S be such that f
n
f uniformly in
S B, and such that B < . (Such a set exists by Egoroffs theorem since S < .) Now throw the set B
0
in
with B (i.e. redene B to be B B
0
). Then,
D
n
:=
_
S
[f
n
g fg[ d =
_
S
[f
n
f[[g[ d
=
_
S\B
[f
n
f[[g[ d +
_
B
[f
n
f[[g[ d
|(f
n
f)
S\B
|
p
|g|
q
+|f
n
f|
p
|g
B
|
q
.
The nal inequality holds because f
n
, f L
p
implies [f
n
f[ L
p
and by H olders inequality. Now, by
Minkowskis inequality, |f
n
f|
p
|f
n
|
p
+|f|
p
, so
D
n
sup[f
n
(x) f(x)[ : x / B(S B)
1/p
|g|
q
+ (|f
n
|
p
+|f|
p
)
__
B
[g[
q
d
_
1/q
.
Now we are free to choose the > 0 above so that
__
B
[g[
q
d
_
1/q
<

2(M +|f|
p
)
holds whenever B < . Also, f
n
f uniformly on S B, so let N be such that
sup[f
N
(x) f(x)[ : x / B <

2(S B)
1/p
|g|
q
.
Then D
N
< /2 +/2. .
(b) The result is not true for p = 1 as the following example shows: Let f
n
= n
[0,1/n]
, n = 1, 2, . . . . First note
that f
n
0 a.e. For if x (0, 1], there exists N > 0 such that 1/N < x and thus f
n
(x) = 0 for all n N.
Therefore, x [0, 1] : f
n
(x) 0 = 0. Now, if g is the constant function g 1, then
_
f
n
g d = 1 for all
n = 1, 2, . . . . Therefore,
_
f
n
g d 1, while
_
fg d =
_
0g d = 0. Finally, note that |f
n
|
1
= n([0,
1
n
]) = 1
for n = 1, 2, . . . , so f
n
satises the hypothesis |f
n
|
1
some constant M. .
30
1.6 2004 April 19 1 REAL ANALYSIS
4. State and prove the closed graph theorem.
5. Prove or disprove:
(a) For 1 p < , let
p
:= x = x
k
[ |x|
p
= (

k=1
[x
k
[
p
)
1/p
< . Then for p ,= 2,
p
is a Hilbert
space.
(b) Let X = (C[0, 1], | |
1
), where the linear space C[0, 1] is endowed with the L
1
-norm: |f|
1
=
_
1
0
[f(x)[ dx.
Then X is a Banach space.
(c) Every real, separable Hilbert space is isometrically isomorphic to
2
.
6. (a) Give a precise statement of some version of Fubinis theorem that is valid for non-negative functions.
(b) Let f, g L
1
(R). (i) Prove that the integral
h(x) =
_
R
f(x t)g(t) dt
exists for almost all x R and that h L
1
(R). (ii) Show that |h|
1
|f|
1
|g|
1
.
7. (a) State the Radon-Nikodym theorem.
(b) Let (X, B, ) be a complete measure space, where is a positive measure dened on the -algebra, B, of
subsets of X. Suppose (X) < . Let S be a closed subset of R and let f L
1
(), where f is an extended
real-valued function dened on X. If
A
E
(f) =
1
(E)
_
E
f d S
for every E B with (E) > 0, prove that f(x) S for almost all x X.
31
1.7 2007 November 16 1 REAL ANALYSIS
1.7 2007 November 16
Notation: R is the set of real numbers and R
n
is n-dimensional Euclidean space. Denote by m Lebesgue measure on
R and m
n
n-dimensional Lebesgue measure. Be sure to give a complete statement of any theorems from analysis that
you use in your proofs below.
1. Let be a positive measure on a measure space X. Assume that E
1
, E
2
, . . . are measurable subsets of X with the
property that for n ,= m, (E
n
E
m
) = 0. Let E be the union of these sets. Prove that
(E) =

n=1
(E
n
)
Solution: Dene F
1
= E
1
, F
2
= E
2
E
1
, F
3
= E
3
(E
1
E
2
), . . . , and, in general,
F
n
= E
n

n1
_
i=1
E
i
(n = 2, 3, . . . ).
If Mis the -algebra of -measurable subsets of X, then F
n
Mfor each n N, since Mis a -algebra. Also,
F
i
F
j
= for i ,= j, and F
1
F
2
F
n
= E
1
E
2
E
n
for all n N. Thus,

_
n=1
F
n
=

_
n=1
E
n
E,
and, by -additivity of ,
(E) = (

_
n=1
F
n
) =

n=1
(F
n
).
Therefore, if we can show (E
n
) = (F
n
) holds for all n N, the proof will be complete.
Now, for each n = 2, 3, . . . ,
F
n
= E
n
(
n1
_
i=1
E
i
)
c
(23)
and
(E
n
) = (E
n
(
n1
_
i=1
E
i
)
c
) +(E
n
(
n1
_
i=1
E
i
)). (24)
Equation (24) holds because

n1
i=1
E
i
is a measurable set for each n = 2, 3, . . . . Finally, note that
E
n
(
n1
_
i=1
E
i
) =
n1
_
i=1
(E
n
E
i
),
which implies
(E
n
(
n1
_
i=1
E
i
))
n1

i=1
(E
n
E
i
),
by -subadditivity. By assumption, each term in the last sum is zero, and therefore, by (23) and (24),
(E
n
) = (E
n
(
n1
_
i=1
E
i
)
c
) = (F
n
) holds for each n = 2, 3, . . . .
For n = 1, we have F
1
= E
1
, by denition. This completes the proof. .
32
1.7 2007 November 16 1 REAL ANALYSIS
2. (a) State a theorem that illustrates Littlewoods Principle for pointwise a.e. convergence of a sequence of functions
on R.
(b) Suppose that f
n
L
1
(m) for n = 1, 2, . . . . Assuming that |f
n
f|
1
0 and f
n
g a.e. as n , what
relation exists between f and g? Make a conjecture and then prove it using the statement in Part (a).
Solution:
(a) I think its generally accepted that the Littlewood principle dealing with a.e. convergence of a sequence of
functions on R is Egoroffs theorem, which is stated below in section A.8.
(b) Conjecture: f = g a.e.
Proof 1:
30
First recall that L
1
convergence implies convergence in measure. That is, if f
n
L
1
(m) and
|f
n
f|
1
0, then f
n
f in measure. (Proof: m(x : [f
n
(x) f(x)[ > )
1

|f
n
f|
1
0.) Next recall
another important theorem
31
which states that if f
n
f in measure then there is a subsequence f
nj
f
n

which converges a.e. to f as j . Combining these two results in the present context (Lebesgue measure on
the real line), we can say the following:
32
If f
n
L
1
(m) and |f
n
f|
1
0 then there is a subsequence
f
nj
f
n
with the property f
nj
(x) f(x) for almost all x R.
Now, if f
n
(x) g(x) for almost all x R, and if B
1
be the set of measure zero where f
n
(x) g(x), then off of
B
1
the sequence f
n
, as well as every subsequence of f
n
, converges to g. Let f
nj
be the subsequence mentioned
above which converges to f almost everywhere. Then
[f(x) g(x)[ [f(x) f
nj
(x)[ +[f
nj
(x) g(x)[. (25)
Dene B
2
= x R : f
nj
(x) f(x). Then the set B = B
1
B
2
has measure zero and, for all x R B,
f
nj
(x) f(x) and f
nj
(x) g(x) . Therefore, by (25), [f(x) g(x)[ = 0 for all x R B. It follows that the
set x R : f
(
x) ,= g(x) B, as a subset of a null set, must itself be a null set (since m is complete). That is,
f = g a.e. and the conjecture is proved. .
Proof 2: First, we claim that if f = g a.e. on [n, n] for every n N, then f = g a.e. in R. To see this, let
B
n
= x [n, n] : f(x) ,= g(x). Then mB
n
= 0 for all n N, so that if B = x R : f(x) ,= g(x), then
B = B
n
and mB

mB
n
= 0, as claimed. Thus, to prove the conjecture, it is enough to show that f = g for
almost every n x n, for an arbitrary xed n N.
Fix n N, and suppose we know that f g L
1
([n, n], m). (This will follow from the fact that f, g
L
1
([n, n], m), which we prove below.) Then, for all > 0 there is a > 0 such that
_
E
[f g[ dm < for
all measurable E [n, n] with mE < . Now apply Egoroffs theorem to nd a set A [n, n] such that
m([n, n] A) < and f
n
g uniformly on A. Then
_
n
n
[f g[ dm =
_
[n,n]\A
[f g[ dm+
_
A
[f g[ dm
+
_
A
[f f
n
[ dm+
_
A
[f
n
g[ dm
+|f f
n
|
1
+mA sup
xA
[f
n
(x) g(x)[,
30
Note that Proof 1, which seems to me the more natural one, doesnt use Egoroffs theorem, so either the examiners were looking for a different
proof, or a different conjecture, or perhaps Egoroffs theorem was not the Littlewood principle they had in mind. In any event, I have found a way
to prove the conjecture which does make use of Egoroffs theorem, and this appears here as Proof 2.
31
Folland [4], theorem 2.30 and its corollary.
32
Perhaps this statement is the version of the Littlewood principle dealing with a.e. convergence that we were meant to cite in part (a).
33
1.7 2007 November 16 1 REAL ANALYSIS
where |f f
n
|
1
0, by assumption, and sup
xA
[f
n
(x) g(x)[ 0 since f
n
g uniformly on A. Since > 0
was arbitrary, it follows that
_
n
n
[f g[ dm = 0 and, for functions f, g L
1
([n, n], m), this implies that f = g
a.e. on [n, n].
It remains to show that f, g L
1
([n, n], m). Its clear that f L
1
since |f|
1
|f f
n
|
1
+ |f
n
|
1
< . To
prove g L
1
([n, n], m) note that f
n
a.e.
g implies lim
n
[f
n
(x)[ = [g(x)[ for almost all x, so by Fatous lemma,
|g|
1
=
_
[g[ dm =
_
lim[f
n
[ dm lim
_
[f
n
[ dm = lim|f
n
|
1
= |f|
1
< .
The last equality holds because, by the triangle inequality, [|f
n
|
1
|f|
1
[ |f
n
f|
1
0. .
3. Let K be a compact subset in R
3
and let f(x) = dist(x, K).
(a) Prove that f is a continuous function and that f(x) = 0 if and only if x K.
(b) Let g = max(1 f, 0) and prove that lim
n
___
g
n
exists and is equal to m
3
(K).
Solution: (a) Dene dist(x, K) = f(x) = inf
kK
[x k[. Clearly, for all k K, f(x) [x k[. Therefore, by
the triangle inequality, for any x, y R
3
,
f(x) [x y[ +[y k[, k K,
and so, taking the inmum over k K on the right,
f(x) [x y[ +f(y). (26)
Similarly,
f(y) [x y[ +f(x). (27)
Obviously, for any given x R
3
, f(x) is nite. Therefore, (26) and (27) together imply that
[f(x) f(y)[ [x y[, x, y R
3
.
Whence f is (Lipschitz) continuous.
Now, if x K, then its clear that f(x) = 0. Suppose x / K; that is, x is in the complement K
c
of K. Since K is
closed, K
c
is open and we can nd an -neighborhood about x fully contained in K
c
, in which case f(x) > . We
have thus proved that f(x) = 0 if and only if x K. .
(b) First observe that f(x) = 0 for all x K, and f(x) > 0 for all x / K. Dene K
1
to be a closed and bounded
set containing K on which f(x) 1. That is, K
1
is the set of points that are a distance of not more than 1 unit
from the set K. In particular K K
1
. Notice that g = max(1 f, 0) = (1 f)
K1
. Also, if x K
1
K,
then 0 1 f(x) < 1, so g
n
0 on the set K
1
K, while on the set K, g
n
= 1 for all n N. Therefore,
g
n

K
. Finally, note that g
n

K1
L
1
(R
3
) so the dominated convergence theorem can be applied to yield
lim
n
___
g
n
=
___

K
= m
3
(K). .
4. Let E be a Borel subset of R
2
.
(a) Explain what this means.
(b) Suppose that for every real number t the set E
t
= (x, y) E [ x = t is nite. Prove that E is a Lebesgue
null set.
34
1.7 2007 November 16 1 REAL ANALYSIS
Solution:
(a) The Borel -algebra of R
2
, which we denote by B(R
2
), is the smallest -algebra that contains the open subsets
of R
2
. The sets belonging to B(R
2
) are called Borel subsets of R
2
.
(b) First observe that if G is a nite subset of R, then G is a Lebesgue null set. That is, mG = 0. In fact, it is easy
to prove that if G is any countable subset, then mG = 0. (Just x > 0 and cover each point x
n
G with a set
E
n
of measure less than 2
n
. Then mG

mE
n
< .)
In problems involving 2-dimensional Lebesgue measure, distinguishing x and y coordinates sometimes claries
things. To wit, let (X, B(X), ) = (Y, B(Y ), ) be two identical copies of the measure space (R, B(R), m), and
represent Lebesgue measure on R
2
by
33
(X Y, B(X) B(Y ), ) = (R
2
, B(R
2
), m
2
).
Our goal is to prove that m
2
E = 0. First note that
m
2
E = ( )(E) =
_
XY

E
d( ).
The integrand
E
is non-negative and measurable (since E is Borel). Therefore, by Tonellis theorem (A.13),
m
2
E =
_
Y
_
X

E
(x, y) d(x) d(y) =
_
X
_
Y

E
(x, y) d(y) d(x). (28)
Now, let
G
t
= y R : (x, y) E and x = t.
This is the so called x-section of E at the point x = t. It is a subset of R, but we can view it as a subset of R
2
by
simply identifying each point y G
t
with the point (t, y) E
t
= (x, y) E : x = t. It follows that, for each
t R, G
t
is a nite subset of R. Therefore, mG
t
= 0. Finally, by (28),
m
2
E =
_
X
_
Y

Gt
(y) d(y) d(t) =
_
X
G
t
d(t) = 0.
since G
t
mG
t
= 0. .
5. Let and be nite positive measures on the measurable space (X, /) such that , and let
d
d(+)
represent the Radon-Nikodym derivative of with respect to +. Show that
0 <
d
d( +)
< 1 a.e. [].
Solution: First note that implies +, so, by the Radon-Nikodym theorem (A.12), there is a unique
f L
1
( +) such that
(E) =
_
E
f d( +) E /.
Indeed, f is the Radon-Nikodym derivative; i.e., f =
d
d(+)
. We want to show 0 < f(x) < 1 holds for -almost
every x X. Since were dealing with positive measures, we can assume f(x) 0 for all x X.
If B
0
= x X : f(x) = 0, then
(B
0
) =
_
B0
f d( +) = 0.
33
The notation B(X) B(Y ) denotes the -algebra generated by all sets A B X Y with A B(X) and B B(Y ). See A.1.7. In
this case, B(X) B(Y ) is the same as B(R
2
).
35
1.7 2007 November 16 1 REAL ANALYSIS
Therefore, (B
0
) = 0, since , which proves that f(x) > 0, []-a.e.
If B
1
= x X : f(x) 1, then
(B
1
) =
_
B1
f d( +) ( +)(B
1
) = (B
1
) +(B
1
).
Since is nite by assumption, we can subtract (B
0
) from both sides to obtain (B
1
) = 0. This proves f(x) < 1,
[]-a.e. .
6.
34
Suppose that 1 < p < and that q = p/(p 1).
(a) Let a
1
, a
2
, . . . be a sequence of real numbers for which the series

a
n
b
n
converges for all real sequences b
n

satisfying the condition

[b
n
[
q
< . Prove that

[a
n
[
p
< .
(b) Discuss the cases of p = 1 and p = . Prove your assertions.
Solution: (a) For each k N, dene T
k
:
q
(N) R by T
k
(b) =

k
n=1
a
n
b
n
, for b
q
(N). Then T
k
is a
family of pointwise bounded linear functionals. That is, each T
k
is a linear functional, and, for each b
q
(N),
there is an M
b
0 such that [T
k
(b)[ M
b
holds for all k N. To see this, simply note that a convergent sequence
of real numbers is bounded, and, in the present case, we have
S
k

k

n=1
a
n
b
n

n=1
a
n
b
n
= x R.
Thus, T
k
(b) = S
k
is a convergent sequence of reals, so, if N N is such that k N implies [S
k
x[ < 1,
and if M

b
is dened to be max[S
k
[ : 1 k N, then, for any k N,
[T
k
(b)[ M
b
maxM

b
, x + 1.
Next note that
q
is a Banach space, so the (Banach-Steinhauss) principle of uniform boundedness implies that
there is a single M > 0 such that |T
k
| M for all k N. In other words,
( M > 0) (b
q
) (k N) [T
k
(b)[ M|b|.
Dene let T(b)

n=1
a
n
b
n
= lim
k
T
k
(b), which exists by assumption. Since [ [ is continuous, we conclude
that lim
k
[T
k
(b)[ = [T(b)[. Finally, since [T
k
(b)[ M|b| for all k N, we have [T(b)[ M|b|. That is T is
a bounded linear functional on
q
(N).
Now, by the Riesz representation theorem, if 1 q < , then any bounded linear functional T

q
is uniquely
representable by some = (
1
,
2
, . . . )
p
as
T(b) =

n=1

n
b
n
. (29)
On the other hand, by denition, T(b) =

n=1
a
n
b
n
, for all b
q
. Since the representation in (29) is unique,
a =
p
. That is,

n
[a
n
[
p
< . .
34
On the original exam this question asked only about the special case p = q = 2.
36
1.7 2007 November 16 1 REAL ANALYSIS
(b) Consider the case p = 1 and q = . First recall that the Riesz representation theorem says that every
T

q
(1 q < ) is uniquely representable by some
p
(where p = q/(q 1), so 1 < p ). That is
p
is the dual of
q
, when 1 q < and p = q/(q 1). However, in the present case we have q = and p = 1
and
1
is not the dual of

. (Perhaps the easiest way to see this is to note that


1
is separable but

is not. For
the collection of a

such that a
n
0, 1, n N, is uncountable and, for any two distinct such sequences
a, b 0, 1
N
, we have |a b|

= 1, so there cannot be a countable base, so

is not second countable, and a


metric space is separable iff it is second countable.)
So we cant use the same method of proof for this case. However, I believe the result still holds by the following
simple argument: Dene b = (b
1
, b
2
, . . . ) by
b
n
= sgn(a
n
) =
_
a
n
/[a
n
[, for a
n
,= 0,
0, for a
n
= 0,
(n N).
Then

n
[a
n
[ =

n
a
n
b
n
converges by the hypothesis, since [b
n
[ 0, 1 implies b

. Therefore, a
1
.
Finally, in case p = and q = 1, the Riesz representation theorem can be applied as in part (a). .
Please email comments, suggestions, and corrections to williamdemeo@gmail.com.
37
2 COMPLEX ANALYSIS
2 Complex Analysis
38
2.1 1989 April 2 COMPLEX ANALYSIS
2.1 1989 April
INSTRUCTIONS: Do at least four problems.
TIME LIMIT: 1.5 hours
35
1. (a) Let U be the unit disk in the complex plane C, U = z C : [z[ < 1, and let f be an analytic function in a
neighborhood of the closure of U. Show that if f is real on all the boundary of U, then f must be constant.
(b) Let u be a real harmonic function in all the complex plane C. Show that if u(z) 0 for all z C, then
u must be constant.
Solution: (a) The hypotheses imply that Imf(e
i
) = 0 for all R. Since f is holomorphic in a neighborhood
of U, the series
f(z) =

n=0
a
n
z
n
converges uniformly on any compact subset of . The unit circle T = z : [z[ = 1 = e
i
: R is one such
compact subset, and here the series is
f(e
i
) =

n=0
a
n
e
in
.
If we write the coefcients as a
n
= c
n
+ib
n
, where c
n
, b
n
R, then we have
a
n
e
in
= (c
n
+ib
n
)[cos(n) +i sin(n)] = [c
n
cos(n) b
n
sin(n)] +i[c
n
sin(n) +b
n
cos(n)].
Thus, by the hypothesis, the series
Imf(e
i
) =

n=0
[c
n
sin(n) +b
n
cos(n)]
converges uniformly to zero for all [0, 2]. Therefore, with the possible exception of c
0
, we have c
n
= b
n
= 0,
for all n, so f c
0
. .
(b) Since C is simply connected, there is a real-valued harmonic conjugate v(z) such that the function f(z) =
u(z) +iv(z) is entire. Now, since u(z) 0, f(z) maps the complex plane into the right half-plane, Ref(z) 0.
It follows immediately from Picards theorem that f must be constant.
36
However, an elementary argument using
only Liouvilles theorem is probably preferable, so let f be as above, and dene g(z) = f(z) + 1. Then g is
entire and maps C into w C : Rew 1. In particular, g(z) is bounded away from zero, so the function
h(z) = 1/g(z) is a bounded entire function. (In fact, [h(z)[ 1.) Therefore, by Liouvilles theorem, h is constant,
so f is constant, so u = Ref is constant. .
2. Let f be an analytic function in the region z : [z[ > 1, and suppose that
lim
z
f(z) = 0.
35
In 1989 there was a single three hour test covering both real and complex analysis. Students were required to do nine problems, with at least
four from each part.
36
Picards theorem states that a non-constant entire function can omit at most one value of C from its range. This is a very powerful theorem, but
if you use it for an easy problem like this one, you might be accused of killing a y with a sledge hammer!
39
2.1 1989 April 2 COMPLEX ANALYSIS
Show that if [z[ > 2, then
1
2i
_
||=2
f()
z
d = f(z). (30)
Solution: By Cauchys formula, if [z[ < R, then
f(z) =
1
2i
_
||=R
f()
z
d
1
2i
_
||=2
f()
z
d. (31)
Note that this holds for all R > [z[ > 2. Fix > 0. Let R

be such that [f()[ < /2 for all [[ = R

and
R

> 2[z[. Then [ z[ > R

/2 for all [[ = R

, so
1
2
_
||=R
[f()[
[ z[
[d[ <
/2
2
2R

/2
= .
Therefore, by (31),

1
2i
_
||=2
f()
z
d +f(z)

< .
This holds for any , which proves (30). .
3.
37
Let U be the open unit disk in C, and let U
+
be the top half of this disk,
U
+
= z C : Imz > 0, [z[ < 1.
Exhibit a one-to-one conformal mapping from U
+
onto U.
Solution: Consider
0
(z) =
1z
1+z
, a linear fractional transformation which takes U onto the right half-plane
= z C : Rez > 0. (This property of
0
can be seen by considering
0
(0) = 1 and
0
() = 1 and
arguing by symmetry.)
Note that
0
(1) = 0 and
0
(x) R, for all x R. Also,
0
is conformal, so it preserves the right angle formed
by the intersection of the circle and the real axis at the point z = 1. Therefore,
0
takes U
+
onto either the rst
quadrant,
+
= z : Imz > 0, or the fourth quadrant,

= z : Imz < 0. To see which, consider


z = i/2.

0
(i/2) =
1 i/2
1 +i/2
=
_
1 i/2
1 +i/2
__
1 i/2
1 i/2
_
=
3 4i
5

.
Thus,
0
: U
+

.
Let
1
(z) = iz, so that
1
:


+
, let
2
(z) = z
2
, so that
2
:
+
Imz > 0, and let
3
(z) = iz,
so that
3
: Imz > 0 = Rez > 0. Finally, note that
1
0
(z) =
0
(z) maps onto U. Putting it all
together, we see that a map satisfying the requirements is
(z) = (
0

3

2

1

0
)(z).
.
37
This problem also appears in April 95 (5) and November 06 (2).
40
2.1 1989 April 2 COMPLEX ANALYSIS
4. Let f
n
be a sequence of analytic functions in the unit disk U, and suppose there exists a constant M such that
_
C
[f
n
(z)[ [dz[ M
for each f
n
and for all circles C lying in U. Prove that f
n
has a subsequence converging uniformly on compact
subsets of U.
Solution: See the solution to problem 7 of November 91.
5. Let Q be a complex polynomial with distinct simple roots at the points a
1
, a
2
, . . . , a
n
, and let P be a complex
polynomial of degree less than that of Q. Show that
P(z)
Q(z)
=
n

k=1
P(a
k
)
Q

(a
k
)(z a
k
)
.
6. Use contour integration and the residue method to evaluate the integral
_

0
cos x
(1 +x
2
)
2
dx.
Solution: Denote the integral by I. Since the integrand is even,
2I =
_

cos x
(1 +x
2
)
2
dx.
Consider the simple closed contour
R
=
R
[R, R], where the trace of
R
is the set Re
i
: 1 ,
oriented counter-clockwise. Note that, if R > 1, then i is inside the region bounded by
R
.
The function
f(z) =
cos z
(1 +z
2
)
2
=
cos z
(z +i)
2
(z i)
2
is holomorphic inside and on
R
, except for a double pole at z = i, where the residue is computed as follows:
Res(f, i) = lim
zi
d
dz
[(z i)
2
f(z)] = lim
zi
d
dz
cos z
(z +i)
2
= lim
zi
(z +i)
2
sin z 2(z +i) cos z
(z +i)
4
=
(2i)
2
sin(i) 4i cos(i)
(2i)
4
=
sin(i) i cos(i)
4
=
ie
ii
4
=
i
4e
.
By the residue theorem, it follows that, for all R > 1,
_

R
f(z) dz = 2i Res(f, i) =

2e
.
41
2.1 1989 April 2 COMPLEX ANALYSIS
It remains to check that
_

R
f(z) dz 0, as R , which will allow us to conclude that
2I =
_

f(x) dx = lim
R
__

R
f(z) dz
_

R
f(z) dz
_
= lim
R
_

R
f(z) dz =

2e
. (32)
Indeed,

R
f(z) dz

R
cos z
(1 +z
2
)
2
dz

1
(R
2
1)
2
(
R
) =
R
(R
2
1)
2
.
This inequality holds for all R > 1, so, letting R , we have
_

R
f(z) dz 0. Therefore, by (32),
I =
_

0
cos x
(1 +x
2
)
2
dx =

4e
.
.
42
2.2 1991 November 21 2 COMPLEX ANALYSIS
2.2 1991 November 21
INSTRUCTIONS: In each of sections A, B, and C, do all but one problem.
TIME LIMIT: 2 hours
SECTION A
(Do 3 of the 4 problems.)
1. Where does the function
f(z) = zRez + zImz + z
have a complex derivative? Compute the derivative wherever it exists.
Solution: Writing f in terms of the real and imaginary parts of z = x +iy, we have
f(x +iy) = (x +iy)x + (x iy)y +x iy
= x
2
+xy +x +i(xy y
2
y)
= u(x, y) +iv(x, y),
where u(x, y) = x
2
+xy +x and v(x, y) = xy y
2
y are the real and imaginary parts of f. Therefore,
u
x
= 2x +y + 1 v
y
= x 2y 1 (33)
u
y
= x v
x
= y. (34)
If f is holomorphic in some region, the Cauchy-Riemann equations (u
x
= v
y
, u
y
= v
x
) must hold there. By (33)
and (34), this requires 2x +y + 1 = x 2y 1 and x = y. Substituting the second equation into the rst yields
y + 1 = 3y 1, or y = 1. Then, since x = y, we must have x = 1. Therefore, f has a complex derivative
at (x, y) = (1, 1), or z = 1 i.
For any region C, we dene the linear functional : H() C by =
1
2
_

x
i

y
_
, and recall that, if
f H(), then the derivative of f is given by f

(z) = (f)(z), z . In the present case,


f
x
= 2x +y + 1 +iy,
f
y
= x +i(x 2y 1).
Therefore, f(x +iy) =
1
2
[(2x +y + 1 +iy) i(x +i(x 2y 1))] =
1
2
[(3x y) +i(y x)], and nally,
f

(1 i) =
1
2
(4 2i) = 2 i.
.
2. (a) Prove that any nonconstant polynomial with complex coefcients has at least one root.
(b) From (a) it follows that every nonconstant polynomial P has the factorization
P(z) = a
N

n=1
(z
n
),
43
2.2 1991 November 21 2 COMPLEX ANALYSIS
where a and each root
n
are complex constants. Prove that if P has only real coefcients, then P has a factorization
P(z) = a
K

k=1
(z r
k
)
M

m=1
(z
2
b
m
z +c
m
),
where a and each r
k
, b
m
, c
m
are real constants.
3. Use complex residue methods to compute the integral
_

0
1
5 + 3 cos
d.
Solution: Let I =
_

0
1
5+3 cos
d. Note that cos is an even function (i.e., cos() = cos ), so
2I =
_

1
5 + 3 cos
d.
For z = e
i
,
cos =
e
i
+e
i
2
=
1
2
(z +
1
z
),
and dz = ie
i
d, from which it follows that
2I =
_
|z|=1
1
5 +
3
2
(z +
1
z
)
dz
iz
=
1
i
_
|z|=1
dz
5z +
3
2
(z
2
+ 1)
=
2
3i
_
|z|=1
dz
z
2
+
10
3
z + 1
.
Let p(z) = z
2
+
10
3
z +1. Then the roots of p(z) are z
1
= 1/3 and z
2
= 3. Only z
1
= 1/3 is inside the circle
[z[ = 1, so the residue theorem implies
2I =
2
3i
2i Res
_
1
p(z)
, z
1
_
.
Now,
1
p(z)
=
1
(z z
1
)(z z
2
)
,
which implies
Res
_
1
p(z)
, z
1
_
= lim
zz1
1
z z
2
=
1

1
3
(3)
=
3
8
.
Therefore,
2I =
2
3i
2i
3
8
=

2
,
so I =

4
. .
44
2.2 1991 November 21 2 COMPLEX ANALYSIS
4. (a) Explain how to map an innite strip (i.e., the region strictly between two parallel lines) onto the unit disk by a
one-to-one conformal mapping.
(b) Two circles lie outside one another except for common point of tangency. Explain how to map the region
exterior to both circles (including the point at innity) onto an innite strip by a one-to-one conformal mapping.
SECTION B
(Do 3 of the 4 problems.)
5.
38
Suppose that f is analytic in the annulus 1 < [z[ < 2, and that there exists a sequence of polynomials converging
to f uniformly on every compact subset of this annulus. Show that f has an analytic extension to all of the disk
[z[ < 2.
Solution: Note that the function f, being holomorphic in the annulus 1 < [z[ < 2, has Laurent series representa-
tion
f(z) =

n=
a
n
(z z
0
)
n
,
converging locally uniformly for 1 < [z[ < 2, where z
0
is any point in the disk [z[ < 2. I claim that a
n
= 0 for all
negative integers n. To see this, rst recall the formula for the coefcients in the Laurent series,
a
n
=
1
2i
_
|z|=R
f(z)
(z z
0
)
n+1
dz, (n Z; 1 < R < 2).
Let p
m
be the sequence of polynomials mentioned in the problemstatement. Of course, p
m
H(C), so Cauchys
theorem implies
_
|z|=R
p
m
(z) dz = 0, and, more generally,
_
|z|=R
p
m
(z)(z z
0
)
n1
dz = 0, (n = 1, 2, . . . ).
Therefore,
[a
n
[ =
1
2

_
|z|=R
f(z)
(z z
0
)
n+1
dz
_
|z|=R
p
m
(z)
(z z
0
)
n+1
dz

1
2
_
|z|=R
[f(z) p
m
(z)[
[z z
0
[
n+1
[dz[. (35)
Finally, p
m
f uniformly on [z[ = R, so (35) implies [a
n
[ = 0 for n = 1, 2, . . . . This proves that f(z) =

n=0
a
n
(z z
0
)
n
, converging locally uniformly in [z[ < 2. Whence f H([z[ < 2). .
6. Let f be analytic in [z[ < 2, with the only zeros of f being the distinct points a
1
, a
2
, . . . , a
n
, of multiplicities
m
1
, m
2
, . . . , m
n
, respectively, and with each a
j
lying in the disk [z[ < 1. Given that g is analytic in [z[ < 2, what
is
_
|z|=1
f

(z)g(z)
f(z)
dz ?
(Verify your answer.)
38
See also: April 96 (8).
45
2.2 1991 November 21 2 COMPLEX ANALYSIS
7. Let f
n
be a sequence of analytic functions in the unit disk D, and suppose there exists a positive constant M
such that
_
C
[f
n
(z)[ [dz[ M
for each f
n
and for every circle C lying in D. Prove that f
n
has a subsequence converging uniformly on compact
subsets of D.
Solution: We must show that T = f
n
is a normal family. If we can prove that T is a locally bounded family
of holomorphic functions that is, T H(D) and, for any compact set K D, there is an M
K
> 0 such that
[f
n
(z)[ M
K
for all z K and all n = 1, 2, . . . then the Montel theorem (corollary 2.2) will give the desired
result.
To show T is locally bounded, it is equivalent to show that, for each point z

D, there is a number M

and a
neighborhood B(z

, r

) D such that [f
n
(z)[ M

for all z B(z

, r

) and all n = 1, 2, . . .. (Why is this


equivalent?)
39
So, x z

D. Let R

> 0 be such that



B(z

, R

) = z C : [z z

[ R

D. Then, for
any z B(z

, R

/2), Cauchys formula gives


[f
n
(z)[
1
2
_
|z|=R
[f
n
()[
[ z[
[d[

1
2
1
R

/2
_
|z|=R
[f
n
()[ [d[

M
R

.
The second inequality holds since [ z

[ = R

and [z z

[ < R

/2 imply [ z[ > R

/2. The last inequality


follows from the hypothesis
_
C
[f
n
(z)[ [dz[ M for any circle C in D. Letting M

=
M
R
, and r

= R

/2, we
have [f
n
(z)[ M

for all z B(z

, r

) and all n = 1, 2, . . . , as desired. .


8. State and prove:
(a) the mean value property for analytic functions
(b) the maximum principle for analytic functions.
39
Answer: If K D is compact, we could select a nite covering of K by such neighborhoods B(z
j
, r
j
) (j = 1, . . . , J) and then
|fn(z)| max
j
M
j
M
K
, for all z K and n = 1, 2, . . . .
46
2.2 1991 November 21 2 COMPLEX ANALYSIS
SECTION C
(Do 2 of the 3 problems.)
9. Let X be a Hausdorff topological space, let K be a compact subset of X, and let x be a point of X not in K. Show
that there exist open sets U and V such that
K U, x V, U V = .
10. A topological space X satises the second axiom of countability. Prove that every open cover of X has a countable
subcover.
11. Let X be a topological space, and let U be a subset of X.
(a) Show that if an open set intersects the closure of Y then it intersects Y .
(b) Show that if Y is connected and if Y Z

Y , then Z is connected.
47
2.3 1995 April 10 2 COMPLEX ANALYSIS
2.3 1995 April 10
Instructions. Work as many of the problems as you can. Each solution should be clearly written on a separate sheet
of paper.
1. Let f(z) =

a
n
z
n
be an entire function.
(a) Suppose that [f(z)[ A[z[
N
+ B for all z C where A, B are nite constants. Show that f is a polynomial
of degree N or less.
(b) Suppose that f satises the condition: [f(z
n
)[ whenever [z
n
[ . Show that f is a polynomial.
Solution: (a) By Cauchys formula, we have
a
n
=
f
(n)
(0)
n!
=
1
2i
_
||=R
f()

n+1
d,
for every R > 0. Therefore,
[a
n
[
1
2
_
||=R
[f()[
[[
n+1
[d[
1
2
AR
N
+B
R
n+1
2R = AR
Nn
+BR
n
.
Again, this holds for every R > 0. Thus, for any n > N and > 0, taking R large enough forces [a
n
[ <
(n = N + 1, N + 2, . . . ). Since was arbitrary, we have a
n
= 0 for all n = N + 1, N + 2, . . . . Therefore,
f(z) =

N
n=0
a
n
z
n
. .
(b) We give three different proofs. The rst is the shortest, but relies on the heaviest machinery.
Proof 1: If we take for granted that any transcendental (i.e. non-polynomial) entire function has an essential singu-
larity at innity, then the Casorati-Weierstrass theorem (see 3 of Nov. 01) implies that, for any complex number
w, there is a sequence z
n
with z
n
and f(z
n
) w as n . Since this contradicts the given hypotheses,
f(z) cannot be a transcendental function. That is, f(z) must be a polynomial. .
Proof 2: Since f H(C), the series f(z) =

a
n
z
n
converges locally uniformly in C. The hypotheses imply that
the function f(1/z) has a pole at z = 0. Let
g(z) = f(1/z) =

n=
b
n
z
n
be the Laurent series expansion of the function g about z = 0. Suppose the pole at z = 0 is of order m. Clearly m
is nite, by the criterion for a pole (i.e., lim
z0
f(1/z) = ). Therefore, we can write
g(z) = f(1/z) =

n=m
b
n
z
n
= b
m
z
m
+b
m+1
z
m+1
+ b
1
z
1
+b
0
+b
1
z + (36)
Now f is entire, so it has the form f(z) =

n=0
a
n
z
n
, which implies that f(1/z) = a
0
+a
1
z
1
+a
2
z
2
+ .
Compared with (36),
a
0
+a
1
z
1
+a
2
z
2
+ = f(1/z) = b
m
z
m
+b
m+1
z
m+1
+ b
1
z
1
+b
0
+b
1
z +
That is, 0 = a
m+1
= a
m+2
= , so
f(z) =
m

n=0
a
n
z
n
.
48
2.3 1995 April 10 2 COMPLEX ANALYSIS
Proof 3: By the hypotheses, there is an R > 0 such that [f(z)[ > 0 for all [z[ > R. Therefore, the zeros of f are
conned to a closed disk D
R
= [z[ R. Since the zeros of f are isolated, there are at most nitely many of
them in any compact subset of C. In particular, D
R
contains only nitely many zeros of f. This proves that f has
only nitely many zeros in C.
Let
1
, . . . ,
N
be the collections of all zeros of f (counting multiplicities). Consider the function
g(z) =
f(z)
(z
1
) (z
N
)
. (37)
This is dened and holomorphic in C
1
, . . . ,
N
, but the
i
s are removable singularities, so g(z) is a nonzero
entire function. In particular, for any R > 0,
min
zD
R
[g(z)[ min
|z|=R
[g(z)[ = > 0,
for some > 0. Therefore, 1/g is a bounded entire function, hence constant, by Liouvilles theorem. What we
have shown is that the left hand side of (37) is constant, and this proves that f(z) is a polynomial. .
Remark: A nice corollary to part (b) is the following:
Corollary 2.1 If f is an injective entire function, then f(z) = az +b for some constants a and b.
The proof appears below in section 2.9.
2. (a) State a form of the Cauchy theorem.
(b) State a converse of the Cauchy theorem.
Solution: (a) See theorem A.16.
(b) See theorem A.18.
3. Let
40
f(z) =

n=0
a
n
z
n
be analytic and one-to-one on [z[ < 1. Suppose that [f(z)[ < 1 for all [z[ < 1.
(a) Prove that

n=1
n[a
n
[
2
1.
(b) Is the constant 1 the best possible?
Solution: (a) This is a special case of the following area theorem:
Theorem 2.1 Suppose f(z) =

n=0
a
n
z
n
is a holomorphic function which maps the unit disk D = [z[ < 1
bijectively onto a domain f(D) = G having area A. Then
A =

n=1
n[a
n
[
2
.
40
On the original exam, the power series representation was given as f(z) =

n=1
anz
n
. However, the problem can be solved without
assuming a
0
= 0 a priori.
49
2.3 1995 April 10 2 COMPLEX ANALYSIS
Proof: The area of the image of D under f is the integral over D of the Jacobian of f. That is,
A =

D
|f

(z)|
2
dxdy.
Compute |f

(z)| by differentiating the power series of f(z) term by term,


f

(z) =

n=1
nanz
n1
.
Next, take the squared modulus,
|f

(z)|
2
=

m,n=1
mnamanz
m1
z
n1
.
This gives,
A =

m,n=1
mnamanz
m1
z
n1
dxdy.
Letting z = re
i
,
A =

m,n=1
mnaman

1
0

2
0
r
m+n1
e
i(mn)
d dr.
Now, for all k = 0, the integral of e
ik
over 0 < 2 vanishes, so the only non-vanishing terms of the series are those for
which m = n. That is,
A = 2

n=1
n
2
|an|
2

1
0
r
2n1
dr =

n=1
n
2
|an|
2
. (38)

To apply this theorem to the problem at hand, note that the hypotheses of the problem imply that f maps the unit
disk bijectively onto its range f(D), which is contained inside D and, therefore, has area less or equal to . This
and (38) together imply

n=1
n
2
[a
n
[
2
,
which gives the desired inequality. .
(b) The identity function f(z) = z satises the given hypotheses and its power series expansion has coefcients
a
1
= 1 and 0 = a
0
= a
2
= a
3
= . This shows that the upper bound of 1 is obtained and is therefore the best
possible. .
4. Let u(z) be a nonconstant, real valued, harmonic function on C. Prove there exists a sequence z
n
with [z
n
[
for which u(z
n
) 0.
Solution: Suppose, by way of contradiction, that there is no such sequence. Then u(z) is bounded away from zero
for all z in some neighborhood of innity, say, [z[ > R, for some R > 0. Since u is continuous, either u(z) > 0
for all [z[ > R, or u(z) < 0 for all [z[ > R. Assume without loss of generality that u(z) > 0 for all [z[ > R.
Since u is continuous on the compact set [z[ R, it attains its minimum on that set. Thus, there is an M > 0
such that M u(z) for all [z[ R. Consider the function U(z) = u(z) + M. By construction, U(z) 0 for
all z C, and U is harmonic in C. But this implies U(z), hence u(z), must be constant.
41
This contradicts the
hypothesis that u(z) be nonconstant and completes the proof. .
41
Recall problem 1(b), April 89, where we proved that a real valued harmonic function u(z) satisfying u(z) 0 for all z C must be constant.
50
2.3 1995 April 10 2 COMPLEX ANALYSIS
5.
42
Find an explicit conformal mapping of the semidisk
H = z : [z[ < 1, Rez > 0
onto the unit disk.
Solution: See the solution to (3) of April 89, or (2) of November 06.
6.
43
Suppose f(z) is a holomorphic function on the unit disk which satises:
[f(z)[ < 1 all [z[ < 1.
(a) State the Schwarz lemma, as applied to f.
(b) If f(0) =
1
2
, how large can [f

(0)[ be?
Solution: (a) See theorem A.22.
(b) Assume f satises the given hypotheses. In particular, f(0) =
1
2
. Consider the map
(z) =
1
2
z
1
z
2
.
This is a holomorphic bijection of the unit disk, with (1/2) = 0. Therefore, g = f satises the hypotheses of
Schwarzs lemma. In particular, [g

(0)[ 1. Since g

(z) =

(f(z))f

(z), we have
1 [g

(0)[ = [

(1/2)[[f

(0)[. (39)
Now,

(z) =

_
1
z
2
_
+
_
1
2
_ _
1
2
z
_
_
1
z
2
_
2
.
Therefore,

(1/2) = 4/3, and it follows from (39) that


[f

(0)[
1
[

(1/2)[
= 3/4.
.
42
This problem also appears in April 89 (3) and November 06 (2).
43
A very similar problem appeared in November 06 (3).
51
2.4 2001 November 26 2 COMPLEX ANALYSIS
2.4 2001 November 26
Instructions. Make a substantial effort on all parts of the following problems. If you cannot completely answer Part
(a) of a problem, it is still possible to do Part (b). Partial credit is given for partial progress. Include as many details as
time permits. Throughout the exam, z denotes a complex variable, and C denotes the complex plane.
1. (a) Suppose that f(z) = f(x+iy) = u(x, y) +iv(x, y) where u and v are C
1
functions dened on a neighborhood
of the closure of a bounded region G C with boundary which is parametrized by a properly oriented, piecewise
C
1
curve . If u and v obey the Cauchy-Riemann equations, show that Cauchys theorem
_

f(z) dz = 0 follows
from Greens theorem, namely
_

P dx +Qdy =
_
G
_
Q
x

P
y
_
dxdy for C
1
functions P and Q. (40)
(b) Suppose that we do not assume that u and v are C
1
, but merely that u and v are continuous in G and
f

(z
0
) = lim
zz0
f(z) f(z
0
)
z z
0
exists at some (possibly only one!) point z
0
G. Show that given any > 0, we can nd a triangular region
containing z
0
, such that if T is the boundary curve of , then

_
T
f(z) dz

=
1
2
L
2
,
where L is the length of the perimeter of .
Hint for (b) Note that part (a) yields
_
T
(az + b) dz = 0 for a, b C, which you can use here in (b), even if you
could not do Part (a). You may also use the fact that

_
T
g(z) dz

L sup[g(z)[ : z T for g continuous on T.


Solution: (a) Let P = u and Q = v in (40). Then, by the Cauchy-Riemann equations,
44
_

u(x, y) dx v(x, y) dy =
_
G
(v
x
+u
y
) dxdy = 0. (41)
Similarly, if P = v and Q = u in Greens theorem, then the Cauchy-Riemann equations imply
_

v(x, y) dx +u(x, y) dy =
_
G
(u
x
v
y
) dxdy = 0. (42)
Next, note that
f(z) dz = [u(x, y) +iv(x, y)] d(x +iy) = u(x, y) dx v(x, y) dy +i[v(x, y) dx +u(x, y) dy].
Therefore, by (41) and (42),
_

f(z) dz =
_

u(x, y) dx v(x, y) dy +i
_

v(x, y) dx +u(x, y) dy = 0.
.
44
These are ux = vy and uy = vx.
52
2.4 2001 November 26 2 COMPLEX ANALYSIS
(b) Suppose u and v are continuous and f

(z) exists at the point z


0
G. Then, for any > 0 there is a > 0 such
that B(z
0
, ) G, and

(z
0
)
f(z) f(z
0
)
z z
0

< , for all [z z


0
[ < .
Pick a triangular region B(z
0
, ) with z
0
, and let T be the boundary. Dene
R(z) = f(z) [f(z
0
) +f

(z
0
)(z z
0
)].
Then, by Cauchys theorem (part (a)),
_
T
[f(z
0
) + f

(z
0
)(z z
0
)] dz = 0, whence
_
T
R(z) dz =
_
T
f(z) dz.
Finally, note that

R(z)
z z
0

(z
0
)
f(z) f(z
0
)
z z
0

< , for all [z z


0
[ < .
Therefore,

_
T
f(z) dz

_
T
R(z) dz

_
T
[R(z)[ [dz[
=
_
T

R(z)
z z
0

[z z
0
[ [dz[

_
T
[z z
0
[ [dz[ rL.
where L denotes the length of the perimeter of (i.e., the length of T), and r denotes the length of one side of T,
which must, of course, be greater than [z z
0
[ for all z T. Also, the length of one side of is surely less than
half the length of the perimeter (i.e., r < L/2). Therefore,

_
T
f(z) dz

1
2
L
2
.
.
2. Give two quite different proofs of the fundamental theorem of algebra, that if a polynomial with complex coef-
cients has no complex zero, then it is constant. You may use independent, well-known theorems and principles
such as Liouvilles theorem, the argument principle, the maximum principle, Rouch es theorem, and/or the open
mapping theorem.
Solution: In the two proofs below, we begin by supposing p(z) is not constant and thus has the form p(z) =
a
0
+ a
1
z + a
2
z
2
+ + a
n
z
n
with a
n
,= 0 for some n 1. Both proofs also rely on the following observation:
If a
j

n
j=0
C with a
n
,= 0, then for all 1 R [z[ < ,

a
0
a
n
z
n
+ +
a
n1
a
n
z
1

[a
0
[
[a
n
[
[z[
n
+ +
[a
n1
[
[a
n
[
[z[
1
n max
0j<n
[a
j
[
[a
n
[
[z[
1
n max
0j<n
[a
j
[
[a
n
[
R
1
.
53
2.4 2001 November 26 2 COMPLEX ANALYSIS
In particular, if we choose
45
R = 1 + 2 n max
0j<n
[a
j
[/[a
n
[, then

a
0
a
n
z
n
+ +
a
n1
a
n
z
1

1/2, for all [z[ R. (43)


Proof 1: Assume p(z) = a
0
+a
1
z+ +a
n
z
n
with a
n
,= 0 for some n 1, and let R = 1+2 n max
0j<n
[a
j
[/[a
n
[,
as above. We claim that
[p(z) a
n
z
n
[ < [a
n
z
n
[, for all [z[ = R. (44)
To see this, check that
[p(z) a
n
z
n
[
[a
n
z
n
[
=

a
0
a
n
z
n
+ +
a
n1
a
n
z
1

< 1, for all [z[ = R.


In fact, (43) implies that the sum is no greater than 1/2, for all [z[ R, which is more than we need. Now (44)
and Rouch es theorem imply that the function g(z) = a
n
z
n
has the same number of zeros in [z[ < R as does the
function p(z). Clearly z = 0 is a zero of g(z) (of multiplicity n). Therefore, p(z) has a zero in [z[ < R. .
Proof 2:
46
Assume p(z) = a
0
+a
1
z + +a
n
z
n
with a
n
,= 0 for some n 1, and consider
[p(z)[ = [a
n
z
n
[

a
0
a
n
z
n
+ +
a
n1
a
n
z
1
+ 1

[a
n
[[z[
n

1 [
n1

j=0
a
j
a
n
z
n+j
[

. (45)
If we choose R = 1 + 2 n max
0j<n
[a
j
[/[a
n
[ as above, then
0

n1

j=0
a
j
a
n
z
n+j

a
0
a
n
z
n
+ +
a
n1
a
n
z
1

1/2, for all [z[ R,


and (45) becomes [p(z)[ [a
n
[[z[
n
/2, for all [z[ R. Therefore, the function f(z) 1/p(z) satises
[f(z)[ =
1
[p(z)[

2
[a
n
[[z[
n
, for all [z[ R.
Now suppose p(z) has no complex zero. Then f(z) H(C). In particular, f(z) is continuous, hence bounded
on the compact set [z[ R. Therefore f(z) is a bounded entire function, so, by Liouvilles theorem, it must be
constant, but then p(z) must be constant. This contradicts our initial assumption and proves that p(z) must have a
complex zero. .
In fact, we have proved a bit more: If p(z) = a
0
+ a
1
z + + a
n
z
n
with a
n
,= 0 for some n 1, and R is
either 1 or R = 2 n max
0j<n
[a
j
[/[a
n
[ (whichever is greater), then p(z) vanishes for some [z[ < R, while for all
[z[ R, [p(z)[ is bounded from below by [a
n
[[z[
n
/2. Thus all the zeros of p(z) are contained in the disk [z[ < R.
45
Note, we add 1 here just to be sure R is safely over 1.
46
Conway [3] (p. 77) presents a similar, but more elegant proof.
54
2.4 2001 November 26 2 COMPLEX ANALYSIS
3. (a) State and prove the Casorati-Weierstrass theorem concerning the image of any punctured disk about a certain
type of isolated singularity of an analytic function. You may use the fact that if a function g is analytic and bounded
in the neighborhood of a point z
0
, then g has a removable singularity at z
0
.
(b) Verify the Casorati-Weierstrass theorem directly for a specic analytic function of your choice, with a suitable
singularity.
Solution:
Theorem 2.2 (Casorati-Weierstrass) If f is a holomorphic function in a region G C except for an essential
singularity at the point z = z
0
, then for any w C there is a sequence z
n
G approaching z
0
such that
f(z
n
) w as n .
Proof: Fix w
0
C and suppose there is no sequence z
n
G approaching z
0
such that f(z
n
) w
0
as n .
Then there is a punctured disk

D
0
B(z
0
, ) z
0
G such that [f(z) w
0
[ > > 0 for all z

D
0
. Dene
g(z) = 1/(f(z) w
0
) on D
0
. Then
limsup
zz0
zD0
[g(z)[ = limsup
zz0
zD0
1
[f(z) w
0
[

1

< .
Thus, by lemma 2.1 (Nov. 06, prob. 1), z
0
is a removable singularity of g(z). Therefore, g(z) H(B(z
0
, )). In
particular, g is continuous and non-zero at z = z
0
, so it is non-zero in a neighborhood B(z
0
,
0
) of z
0
. Therefore,
f(z)w
0
= 1/g(z) is holomorphic in B(z
0
,
0
), which implies that the singularity of f(z) at z = z
0
is removable.
This contradiction proves the theorem. .
(b) Consider f(z) = e
z
. This function has an essential singularity at , and, for every horizontal strip,
S

= x +iy : x R, y < + 2,
of width 2, f(z) maps S

onto C 0. (In particular, f(z) comes arbitrarily close to every w C.) Now let
A
R
= z C : [z[ > R be any neighborhood of . There is clearly a strip S

contained in A
R
(e.g., with
= R + 1). Therefore, f(z) = e
z
maps points in A
R
to points arbitrarily close (in fact equal when w ,= 0) to all
points w C. .
4. (a) Dene : [0, 2] C by (t) = sin(2t) + 2i sin(t). This is a parametrization of a gure 8 curve, traced
out in a regular fashion. Find a meromorphic function f such that
_

f(z) dz = 1. Be careful with minus signs and


factors of 2i.
(b) From the theory of Laurent expansions, it is known that there are constants a
n
such that, for 1 < [z[ < 4,
1
z
2
5z + 4
=

n=
a
n
z
n
.
Find a
10
and a
10
by the method of your choice.
Solution: (a) Let G be the region whose boundary is the curve , and suppose f(z) H(C) except for isolated
singularities at the points z
1
, . . . , z
n
G. By the residue theorem,
_

f(z) dz = 2i
n

j=1
Res(f, z
j
).
55
2.4 2001 November 26 2 COMPLEX ANALYSIS
Therefore, if we were to nd a function f(z) H(C) with exactly two isolated singularities in G (e.g., at z
1
= i
and z
2
= i), and such that Res(f, z
j
) =
i
4
, then
_

f(z) dz = 2i

j
Res(f, z
j
) = 2i
_
i
4
+
i
4
_
= 1,
and the problem would be solved. Clearly,
f(z) =
i
4
_
1
z i

1
z +i
_
=
1
2i
z
z
2
+ 1
is such a function. .
(b) Expand the function in partial fractions:
1
z
2
5z + 4
=
1
(z 4)(z 1)
=
1/3
z 4

1/3
z 1
.
Then, note that
1/3
z 4
=
1
3
1
4(1 z/4)
=
1
12

n=0
_
z
4
_
n
converges for [z[ < 4, while
1/3
z 1
=
1
3
1
z(1 1/z)
=
1
3z

n=0
z
n
converges for [z[ > 1. Therefore,
1
z
2
5z + 4
=
1
3
1

n=
z
n

1
12

n=0
_
1
4
_
n
z
n
, for 1 < [z[ < 4.
a
10
=
1
3
and a
10
=
1
12
_
1
4
_
10
.
.
5. (a) Suppose that f is analytic on a region G C and z C : [z a[ R G. Show that if [f(z)[ M for
all z with [z a[ = R, then for any w
1
, w
2
z C : [z a[
1
2
R, we have
[f(w
1
) f(w
2
)[
4M
R
[w
1
w
2
[
(b) Explain how Part (a) can be used with the Arzela-Ascoli theorem to prove Montels theorem asserting the
normality of any locally bounded family F of analytic functions on a region G.
Solution: (a) By Cauchys formula (A.14), if w is any point in the disk [w a[ < R, then
f(w) =
1
2i
_
|a|=R
f()
w
d.
56
2.4 2001 November 26 2 COMPLEX ANALYSIS
In particular, if w
1
, w
2
are any two points inside the half-disk [w a[ < R/2 (see gure 1), then
f(w
1
) f(w
2
) =
1
2i
_
|a|=R
_
f()
w
1

f()
w
2
_
d
=
w
1
w
2
2i
_
|a|=R
f()
( w
1
)( w
2
)
d.
R
.
w
.
a
W
1
W
2
.
.
R/2
.

Figure 1: Note that, if is any point on the outer radius, [ a[ = R, and if w is any point in the disk [wa[ < R/2,
then [ w[ > R/2.
Now, for all on the outer radius in gure 1, it is clear that [ w
1
[ > R/2 and [ w
2
[ > R/2. Therefore,
[f(w
1
) f(w
2
)[
[w
1
w
2
[
2
_
|a|=R
[f()[
(R/2)
2
[d[

[w
1
w
2
[
2
sup

[f()[
R
2
/4
()

4M
R
[w
1
w
2
[,
where denotes the positively oriented circle : [ a[ = R, and () denotes its length, 2R. .
(b)
47
We must explain how part (a) can be used with the Arzela-Ascoli theorem to prove Montels theorem asserting
the normality of any locally bounded family T H(G).
Theorem 2.3 (Arzela-Ascoli) Let T C(G, S) be a family of continuous functions from an open set G C into
a metric space (S, d). Then T is a normal family if and only if
(i) T is equicontinuous on each compact subset of G, and
(ii) for each z G, the set f(z) : f T is contained in a compact subset of S.
47
The best treatment of normal families and the Arzela-Ascoli theorem is Ahlfors [1].
57
2.4 2001 November 26 2 COMPLEX ANALYSIS
Recall that a family T of functions is called locally bounded on G iff for all compact K G there is a constant
M
K
such that [f(z)[ M
K
for all f T and z K.
Corollary 2.2 (little Montel theorem) Assume the set-up of the Arzela-Ascoli theorem, and suppose S = C and
T H(G). Then T is a normal family if and only if it is locally bounded.
Because of the way the problem is stated, it is probably enough to prove just one direction of Montels theorem;
i.e., local boundedness implies normality. For a proof of the other direction, see Conway [3], page 153.
Let S = C in the Arzela-Ascoli theorem. In that case, K C is compact if and only if K is closed and bounded.
Therefore, if T is locally bounded, condition (ii) of the theorem is clearly satised. To check that local boundedness
also implies condition (i), we use part (a).
It sufces (why?)
48
to prove that for any a G there is a neighborhood B(a, r) in which T is equicontinuous with
equicontinuity constant
49
. So, x a G and > 0, and let

B(a, R) G. Then, by local boundedness, there is
an M > 0 such that [f(z)[ M for all z

B(a, R) and all f T. Therefore, by part (a),
[f(w
1
) f(w
2
)[
4M
R
[w
1
w
2
[, for all w
1
, w
2
[w a[ R/2.
If =
R
4M
and r = R/2, then [f(w
1
) f(w
2
)[ < whenever w
1
, w
2
B(a, r) and [w
1
w
2
[ < . Therefore,
T is equicontinuous in B(a, r). We have thus shown that local boundedness implies conditions (i) and (ii) of the
Arzela-Ascoli theorem and thereby implies normality. .
48
Answer: If, instead of a single point a G, we are given a compact set K G, then there is a nite cover {B(a
j
, r
j
) : j = 1, . . . , n} by
such neighborhoods with equicontinuity constants
1
, . . . , n. Then, = min
j

j
, is a single equicontinuity constant that works for all of K.
49
The careful reader might note the distinction between this type of uniform equicontinuity, which is taken for granted in complex analysis
texts, e.g., Ahlfors [1] and Rudin [8], and the pointwise equicontinuity discussed in topology books like the one by Munkres [5]. To make peace
with this apparent discrepancy, check that the two notions coincide when the set on which a family of functions is declared equicontinuous is
compact.
58
2.5 2004 April 19 2 COMPLEX ANALYSIS
2.5 2004 April 19
Instructions. Use a separate sheet of paper for each new problem. Do as many problems as you can. Complete
solutions to ve problems will be considered as an excellent performance. Be advised that a few complete and well
written solutions will count more than several partial solutions.
Notation. D(z
0
, R) = z C : [z z
0
[ < R R > 0. For an open set G C, H(G) will denote the set of
functions which are analytic in G.
1. Let be a rectiable curve and let C(

). (That is, is a continuous complex function dened on the trace,

, of .)
Let F(z) =
_

(w)
(wz)
dw, z C

.
Prove that F

(z) =
_

(w)
(wz)
2
dw, z C

, without using Leibnizs Rule.


2. (a) State the Casorati-Weierstrass theorem.
(b) Evaluate the integral
I =
1
2i
_
|z|=R
(z 3) sin
_
1
z + 2
_
dz where R 4.
3. Let f(z) be an entire function such that f(0) = 1, f

(0) = 0 and
0 < [f(z)[ e
|z|
for all z C
Prove that f(z) = 1 for all z C.
Solution: I know of two ways to prove this. One can be found in Rudins Functional Analysis ([9], p. 250). The
other goes as follows:
50
By the Hadamard factorization theorem (see, e.g., [11]), an entire function f with zeros at a
n
C 0 and m
zeros at z = 0 has the form
f(z) = e
P(z)
z
m

n=0
_
1
z
a
n
_
e
z/an
, (46)
where P(z) is a polynomial of degree , the order of growth, and k < k + 1. For the function in question,
we have [f(z)[ > 0 so a
n
= and m = 0. Also, since [f(z)[ e
|z|
, the order of growth is = 1, which implies
that P(z) is a polynomial of degree 1. Therefore, (46) takes the simple form,
f(z) = e
Bz+C
,
for some constants B, C. We are given that f(0) = 1 and f

(0) = 0, so e
C
= 1, and f

(0) = Be
C
= B = 0. It
follows that f(z) = 1. .
50
This proof came to me by sheer lucky coincidence I worked on this exam after having just read a beautiful treatment of the Hadamard
factorization theorem in Stein and Sharkachis new book [11]. If you need convincing that this theorem is worth studying, take a look at how easily
it disposes of this otherwise challenging exam problem. Also, Stein and Sharkachi seem to have set things up just right, so that the theorem is very
easy to apply.
59
2.6 2006 November 13 2 COMPLEX ANALYSIS
2.6 2006 November 13
Notation: C is the set of complex numbers, D = z C : [z[ < 1 is the open unit disk,
+
and

are the upper


and lower half-planes, respectively, and, given an open set G C, H(G) is the set of holomorphic functions on G.
1. (a) Suppose that f H(D 0) and that [f(z)[ < 1 for all 0 < [z[ < 1. Prove that there is F H(D) with
F(z) = f(z) for all z D 0.
(b) State a general theorem about isolated singularities for holomorphic functions.
Solution:
Lemma 2.1 Suppose G C is an open set and f is holomorphic in G except for an isolated singularity at z
0
G.
If
limsup
zz0
zG
[f(z)[ < ,
then z
0
is a removable singularity and f may be extended holomorphically to all of G.
Proof: Under the stated hypotheses, there is an > 0 and an M > 0 such that the deleted neighborhood B
o

z C : 0 < [z z
0
[ is contained in G and such that [f(z)[ M for all z B
o
.
Let
f(z) =

n=
a
n
(z z
0
)
n
be the Laurent expansion of f for z B
o
, where
a
n
=
1
2i
_
C
f()
( z
0
)
n+1
d.
Here C denotes the positively oriented circle [ z
0
[ = . Changing variables,
= z
0
+e
i
d = i e
i
d
the coefcients are
a
n
=
1
2 i
_
2
0
f(z
0
+e
i
)
(z
0
+e
i
z
0
)
n+1
ie
i
d.
Therefore,
[a
n
[
1
2
_
2
0
M

n+1
d[[ =
M

n
,
which makes it clear that, if n < 0, then [a
n
[ can be made arbitrarily small, by choosing a sufciently small . This
proves that a
n
= 0 for negative n, and so
f(z) =

n=0
a
n
(z z
0
)
n
.
Thus, f H(G). .
The lemma solves part (a) and is also an example of a general theorem about isolated singularities of holomorphic
functions, so it answers part (b). Here is another answer to part (b):
60
2.6 2006 November 13 2 COMPLEX ANALYSIS
Theorem 2.4 (Criterion for a pole) Let G C be open. and suppose f(z) is holomorphic for all z G except
for an isolated singularity at z = z
0
G. Then
(i) z
0
is a pole of f if and only if lim
zz0
[f(z)[ = ;
(ii) if m > 0 is the smallest integer such that lim
zz0
[(z z
0
)
m
f(z)[ remains bounded, then z
0
is a pole of
order m.
.
2. (a) Explicitly construct, through a sequence of mappings, a one-to-one holomorphic function mapping the disk D
onto the half disk D
+
.
(b) State a general theorem concerning one-to-one mappings of D onto domains C.
Solution: (a)
51
Let
0
(z) =
1z
1+z
. Our strategy will be to show that
0
maps the fourth quadrant onto D
+
,
and then to construct a conformal mapping, f, of the unit disk onto the fourth quadrant. Then the composition

0
f will have the desired properties.
Consider the boundary of the rst quadrant. Note that
0
maps the real line onto itself. Furthermore,
0
takes 0 to
1 and takes to -1. Since
0
(1) = 0, we see that the positive real axis (0, ) is mapped onto the segment (1, 1).
Now, since
0
maps the right half-plane P
+
onto the unit disk, it must map the boundary of P
+
(i.e., the imaginary
axis) onto the boundary of the unit disk. Thus, as 0 1 and 1, the positive imaginary axis is mapped to
either the upper half-circle or the lower half-circle, and similarly for the negative imaginary axis. Checking that

0
(i) = i, it is clear that the positive imaginary axis is mapped to the lower half-circle e
i
: < < 0.
Therefore, in mapping the right half-plane onto the unit disk,
0
maps the rst quadrant to the lower half-disk
D

, and must therefore map the fourth quadrant to the upper half-disk. That is,
0
: Q
4
D
+
, where
Q
4
= z C : Rez > 0, Imz < 0.
Next construct a mapping of the unit disk onto the fourth quadrant as follows: If
1
(z) = iz, then
1

0
: D

+
. Let
2
(z) = z
1/2
be a branch of the square root function on
+
. Then
2
maps
+
onto the rst quadrant,
Q
1
= z C : Rez > 0, Imz > 0. Finally, let
3
(z) = e
i/2
z = iz, which takes the rst quadrant to the
fourth quadrant. Then, since all of the mappings are conformal bijections, f =
3

2

1

0
is a conformal
bijection of D onto Q
4
. Therefore,
0
f is a conformal bijection of D onto D
+
. .
(b) (Riemann)
52
Let C be a simply connected region such that ,= C. Then is conformally equivalent
to D. That is, there is a conformal bijection, , of onto the unit disk. Moreover, if we specify that a particular
z
0
must be mapped to 0, and we specify the value of arg (z
0
), then the conformal mapping is unique.
3.
53
(a) State the Schwarz lemma.
(b) Suppose that f H(
+
) and that [f(z)[ < 1 for all z
+
. If f(i) = 0 how large can [f

(i)[ be? Find the


extremal functions.
Solution: (a) See theorem A.22.
51
This problem also appears in April 89 (3) and April 95 (5).
52
Look up the precise statement of the Riemann mapping theorem.
53
A very similar problem appeared in April 95 (6).
61
2.6 2006 November 13 2 COMPLEX ANALYSIS
(b) In order to apply the Schwarz lemma, map the disk to the upper half-plane with the M oebius map : D
+
dened by
(z) = i
1 z
1 +z
.
Then, (0) = i. Therefore, the function g = f : D


+
f
D satises [g(z)[ 1 and g(0) = f((0)) =
f(i) = 0. By Schwarzs lemma, then, [g

(0)[ 1. Finally, observe that g

(z) = f

((z))

(z), and then check


that

(0) = 2i. Whence, g

(0) = f

((0))

(0) = f

(i)(2i), which implies 1 [g

(0)[ = 2[f

(i)[. Therefore
[f

(i)[ 1/2. .
4. (a) State Cauchys theorem and its converse.
(b) Suppose that f is a continuous function dened on the entire complex plane. Assume that
(i) f H(
+

)
(ii) f( z) = f(z) all z C.
Prove that f is an entire function.
Solution: (a) See theorems A.16 and A.18.
(b) See Marsden and Hoffman.
5. (a) Dene what it means for a family T H() to be a normal family. State the fundamental theorem for normal
families.
(b) Suppose f H(
+
) and [f(z)[ < 1 all z
+
. Suppose further that
limt 0+f(it) = 0.
Prove that f(z
n
) 0 whenever the sequence z
n
0 and z
n
where
= z
+
: [Rez[ Imz.
Hint. Consider the functions f
t
(z) = f(tz) where t > 0.
Solution: (a) Let be an open subset of the plane. A family T of functions in is called a normal family if
every sequence of functions in T has a subsequence which converges locally uniformly in . (The same denition
applies when the family T happens to be contained in H().)
54
I think of the Arzela-Ascoli theorem as the fundamental theorem for normal families. However, since the examiners
asked specically about the special case when T is a family of holomorphic functions, they probably had in mind
the version of Montels theorem stated below, which is an easy consequence of the Arzela-Ascoli theorem.
55
Theorem 2.5 (Arzela-Ascoli) Let T C(, S) be a family of continuous functions from an open set C into
a metric space (S, d). Then T is a normal family if and only if
(i) T is equicontinuous on each compact subset of , and
(ii) for each z , the set f(z) : f T is contained in a compact subset of S.
54
Despite the wording of the problem, the family need not satisfy F H() in order to be normal.
55
Problem 5 (b) of the November 2001 exam asks for a proof of Montels theorem using the Arzela-Ascoli theorem.
62
2.6 2006 November 13 2 COMPLEX ANALYSIS
Recall that a family T of functions is called locally bounded on iff for all compact K there is a constant
M
K
such that [f(z)[ M
K
for all f T and z K.
Corollary 2.3 (little Montel theorem
56
) Assume the set-up of the Arzela-Ascoli theorem, and suppose S = C
and T H(). Then T is a normal family if and only if it is locally bounded.
(b) Fix a sequence z
n
with z
n
0 as n . We must prove f(z
n
) 0. Dene f
n
(z) = f([z
n
[z).
Then, since z [z
n
[z , we have
[f
n
(z)[ = [f([z
n
[z)[ < 1, for all z and n N.
Therefore, T is a normal family in . Also note that each f
n
is holomorphic in since f(tz) H() for any
constant t > 0. Thus, T is a normal family of holomorphic functions in .
Let g be a normal limit of f
n
; i.e., there is some subsequence n
k
such that, as k , f
n
k
g locally uniformly
in .
Consider the point z = i. Since f(it) 0 as t 0,
g(i) = lim
k
f
n
k
(i) = lim
k
f([z
n
k
[i) = 0.
In fact, for any point z = iy with y > 0, we have g(z) = 0. Since g is holomorphic in , the identity theorem
implies that g 0 in .
Next, consider
f
n
_
z
n
[z
n
[
_
= f
_
[z
n
[
z
n
[z
n
[
_
= f(z
n
). (47)
The numbers z
n
/[z
n
[ lie in the compact set = z : [z[ = 1. Since f
n
k
g uniformly in , for any > 0,
there is a K > 0 such that [f
n
k
(z) g(z)[ = [f
n
k
(z)[ < , for all k K and all z . That is,
lim
k
sup[f
n
k
(z)[ : z lim
k
|f
n
k
|

= 0,
and, since z
n
k
/[z
n
k
[ ,

f
n
k
_
z
n
k
[z
n
k
[
_

|f
n
k
|

.
lim
k
f
n
k
_
z
n
k
[z
n
k
[
_
= 0.
By (47), then, lim
k
f(z
n
k
) = 0.
Finally, recall that f(z
n
) 0 iff every subsequence z
nj
has a further subsequence z
nj
k
such that f(z
nj
k
) 0,
as k . Now, if z
nj
is any subsequence, then f(z
nj
) is a normal family, and, repeating the argument above,
there is, indeed, a further subsequence z
nj
k
such that f(z
nj
k
) 0. This completes the proof. .
Remark: In the last paragraph, we made use of the fact that a sequence converges to zero iff any subsequence has,
in turn, a further subsequence that converges to zero. An alternative concluding argument that doesnt rely on this
result, but proceeds by way of contradiction, runs as follows: Assume we have already shown lim
k
f(z
n
k
) = 0,
as above, and suppose f(z
n
) does not converge to 0 as n . Then there is a > 0 and a subsequence z
nj

such that [f(z


nj
)[ > for all j N. Relabel this subsequence z
n
. Then f(z
n
) is itself a normal family and
we can repeat the argument above to get a further subsequence z
n
k
with lim
k
f(z
n
k
) = 0. This contradicts
the assumption that [f(z
n
)[ > for all n N. Therefore, f(z
n
) 0, as desired.
63
2.7 2007 April 16 2 COMPLEX ANALYSIS
2.7 2007 April 16
Notation: C is the set of complex numbers, D = z C : [z[ < 1, and, for any open set G C, H(G) is the set of
holomorphic functions on G.
1. Give the Laurent series expansion of
1
z(z1)
in the region A z C : 2 < [z + 2[ < 3.
Solution:
f(z) =
1
z(z 1)
=
1 z +z
z(z 1)
=
1
z 1

1
z
.
Let u = z + 2. Then z = u 2 and A = u C : 2 < [u[ < 3. Therefore,
1
z
=
1
u 2
=
1
u
1
(1 2/u)
=
1
u

n=0
_
2
u
_
n
converges for [u[ > 2 and, substituting u = z + 2 in the last expression, we have
1
z
=
1
z + 2

n=0
_
2
z + 2
_
n
=

n=0
2
n
(z + 2)
n1
=
1

n=
_
1
2
_
n+1
(z + 2)
n
,
converging for 2 < [z + 2[. Next, consider that
1
z 1
=
1
u 3
=
1
3(1 u/3)
=
1
3

n=0
_
u
3
_
n
converges for [u[ < 3 and, substituting u = z + 2 in the last expression, we have
1
z 1
=

n=0
_
1
3
_
n+1
(z + 2)
n
,
converging for [z + 2[ < 3. Therefore,
f(z) =
1
z 1

1
z
=

n=0
_
1
3
_
n+1
(z + 2)
n

n=
_
1
2
_
n+1
(z + 2)
n
,
for z A. .
2. (i) Prove: Suppose that for all z D and all n N we have that f
n
is holomorphic in D and [f
n
(z)[ < 1. Also
suppose that lim
n
Imf
n
(x) = 0 for all x (1, 0). Then lim
n
Imf
n
(1/2) = 0.
(ii) Give a complete statement of the convergence theorem that you use in part (2i).
Solution: (i)
(ii)
3. Use the residue theorem to evaluate
_

1
1+x
4
dx.
64
2.7 2007 April 16 2 COMPLEX ANALYSIS
Solution: Note that
f(z) =
1
1 +x
4
=
1
(z
2
+i)(z
2
i)
=
1
(z +e
i/4
)(z e
i/4
)(z +e
i3/4
)(z e
i3/4
)
,
which reveals that the poles of f in the upper half plane are at e
i/4
and e
i3/4
. Let
R
be the contour shown in
the gure below; i.e.,
R
= g(R) [R, R], where R > 1. Then, by the residue theorem,
_

R
f(z)dz = 2i
_
Res(f, e
i/4
) + Res(f, e
i3/4
)
_
. (48)
The other two poles of f are in the lower half-plane, so both e
i/4
and e
i3/4
are simple poles. Therefore,
Res(f, e
i/4
) = lim
ze
i/4
(z e
i/4
)f(z) =
1
2e
i/4
(e
i/4
e
i3/4
)(e
i/4
+e
i3/4
)
=
1
4
ie
i/4
,
Res(f, e
i3/4
) = lim
ze
i3/4
(z e
i3/4
)f(z) =
1
2e
i3/4
(e
i3/4
e
i/4
)(e
i3/4
+e
i/4
)
=
1
4
ie
i3/4
.
Plugging these into (48) yields
_

R
f(z)dz = 2i
_
1
4
ie
i3/4

1
4
ie
i/4
_
=

2
(e
i/4
e
i3/4
) =

2
.
It remains to show
lim
R

_
g(R)
f(z)dz

= 0.
Changing variables via z = Re
i
(0 ),

_
g(R)
f(z)dz

_

0
iRe
i
1 + (Re
i
)
4

R
R
4
1
0, as R .
.
4. Present a function f that has all of the following properties: (i) f is one-to-one and holomorphic on D. (ii)
f(z) : z D = w C : Rew > 0 and Imw > 0. (iii) f(0) = 1 +i.
65
2.7 2007 April 16 2 COMPLEX ANALYSIS
Solution: First consider
57

1
(z) =
1z
1+z
, which maps D onto the right half-plane P
+
= z C : Rez > 0.
Let
2
(z) = e
i/2
z = iz, which maps P
+
onto the upper half-plane
+
= z C : Imz > 0. Next,
let
3
(z) = z
1/2
be a branch of the square root function on
+
. Then
3
maps
+
onto the rst quadrant
Q
1
= z C : 0 < arg(z) < /2.
The function =
3

2

1
satises the rst two conditions, so we check whether it satises condition (iii):

1
(0) = 1 (
2

1
)(0) =
2
(1) = i (
3

2

1
)(0) =
3
(i) =
1 +i

2
so apparently were off by a factor of

2. This is easy to x: let
4
(z) =

2z. Then the holomorphic function


f
4
maps D bijectively onto Q
1
and f(0) = 1 +i, as desired. .
5. (i) Prove: If f : D D is holomorphic and f(1/2) = 0, then [f(0)[ 1/2.
(ii) Give a complete statement of the maximum modulus theorem that you use in part (i).
Solution: (i) Dene (z) =
1/2z
1z/2
. This is a holomorphic bijection
58
of

D onto

D. Therefore, g = f H(D),
[g(z)[ 1 for all z D, and g(0) = f((0)) = f(1/2) = 0. Thus g satises the hypotheses of Schwarzs lemma
(theorem A.22), which allows us to conclude the following:
(a) [g(z)[ [z[, for all z D, and
(b) [g

(0)[ 1,
with equality in (a) for some z D or equality in (b) iff g(z) = e
i
z for some constant R. By condition (a),
1/2 [g(1/2)[ = [f((1/2))[ = [f(0)[.
.
(ii) In part (i) I used Schwarzs lemma, a complete statement of which appears in the appendix (theorem A.22).
This is sometimes thought of as a version of the maximum modulus principle since it is such an easy corollary of
what is usually called the maximum modulus principle. Here is a complete statement of the latter:
(max modulus principle, version 1)
Suppose G C is open and f H(G) attains its maximum modulus at some point a G. Then f is constant.
That is, if there is a point a G with [f(z)[ [f(a)[ for all z G, then f is constant.
59
6. Prove: If G is a connected open subset of C, any two points of G can be connected by a parametric curve in G.
57
This is my favorite M oebius map. Not only does it map the unit disk onto the right half-plane, but also it maps the right half-plane onto the
unit disk. This feature makes
1
an extremely useful tool for conformal mapping problems, where youre frequently required to map half-planes to
the unit disk and vice-versa. Another nice feature of this map is that
1
1
=
1
. (Of course this must be the case if
1
is to have the rst feature.)
Also note that, like all linear fractional transformations,
1
is a holomorphic bijection of C. Therefore, if
1
is to map the interior of the unit disk
to the right half-plane, it must also map the exterior of the unit disk to the left half-plane.
58
See Rudin [8] page 254-5 (in particular, theorem 12.4) for a nice discussion of functions of the form (z) =
z
1 z
. In addition to 12.4,
sec. 12.5 and theorem 12.6 are popular exam questions.
59
There are a couple of other versions of the maximum modulus principle you should know, though for most problems on the comprehensive
exams, the version above usually sufces. The other two versions are stated and proved clearly and concisely in Conway [3], but they also appear
as theorems A.20 and A.21 of the appendix.
66
2.7 2007 April 16 2 COMPLEX ANALYSIS
Solution: First, recall that if A G C, then A is said to be open relative to G, or simply open in G, if for any
a A there is a neighborhood B(a, ) = z C : [z a[ < such that B(a, ) G A.
60
Next, recall that a subset G C is connected iff the only subsets of G that are both open and closed relative to G
are the empty set and G itself. Equivalently, if there exist non-empty disjoint subsets A, B G that are open in G
and have the property G = A B, then G is not connected, or disconnected.
61
Now, suppose G is a connected open subset of C. Fix z
0
G and let G be the subset of points that can
be connected to z
0
by a parametric curve in G. Since G is open, B(z
0
, ) G for some > 0, and clearly
B(z
0
, ) . In particular, ,= . If we can show is both open and closed in G, then it will follow by
connectedness that = G, and the problem will be solved.
( is open) Let w be connected to z
0
by a parametric curve G. Since G is open, > 0 such that
B(w, ) G. Clearly any w
1
B(w, ) can be connected to z
0
by a parametric curve (from w
1
to w, then from w
to z
0
via ) that remains in G. This proves that B(w, ) , so is open.
( is closed) We show G is open (and thus, in fact, empty). If z G , then, since G is open, > 0
such that B(z, ) G. We want B(z, ) G . This must be true since, otherwise, there would be a point
z
1
B(z, ) which could be connected to both z and z
0
by parametric curves in G. But then a parametric
curve in G connecting z to z
0
could be constructed, which would put z in a contradiction.
We have thus shown that is both open and closed in G, as well as non-empty. Since G is connected, = G. .
60
For example, the set A = [0, 1], although closed in C, is open in G = [0, 1] {2}.
61
To see the equivalence note that, in this case, A is open in G, as is A
c
= G\ A = B, so A is both open and closed in G. Also it is instructive
to check, using either denition, that G = [0, 1] {2} is disconnected.
67
2.8 2007 November 16 2 COMPLEX ANALYSIS
2.8 2007 November 16
Do as many problems as you can. Complete solutions (except for minor aws) to 5 problems would be considered an
excellent performance. Fewer than 5 complete solutions may still be passing, depending on the quality.
1. Let G be a bounded open subset of the complex plane. Suppose f is continuous on the closure of G and analytic
on G. Suppose further that there is a constant c 0 such that [f[ = c for all z on the boundary of G. Show that
either f is constant on G or f has a zero in G.
2. (a) State the residue theorem.
(b) Use contour integration to evaluate
_

0
x
2
(x
2
+ 1)
2
dx.
Important: You must carefully: specify your contours, prove the inequalities that provide your limiting arguments,
and show how to evaluate all relevant residues.
3. (a) State the Schwarz lemma.
(b) Suppose f is holomorphic in D = z : [z[ < 1 with f(D) D. Let f
n
denote the composition of f with
itself n times (n = 2, 3, . . . ). Show that if f(0) = 0 and [f

(0)[ < 1, then f


n
converges to 0 locally uniformly
on D.
4. Exhibit a conformal mapping of the region common to the two disks [z[ < 1 and [z 1[ < 1 onto the region inside
the unit circle [z[ = 1.
5. Let f
n
be a sequence of functions analytic in the complex plane C, converging uniformly on compact subsets of
Cto a polynomial p of positive degree m. Prove that, if n is sufciently large, then f
n
has at least mzeros (counting
multiplicities).
Do not simply refer to Hurwitzs theorem; prove this version of it.
6. Let (X, d) be a metric space.
(a) Dene what it means for a subset K X to be compact.
(b) Prove (using your denition in (a)) that K X is compact implies that K is both closed and bounded in X.
(c) Give an example that shows the converse of the statement in (b) is false.
Please email comments, suggestions, and corrections to williamdemeo@gmail.com.
68
2.9 Some problems of a certain type 2 COMPLEX ANALYSIS
2.9 Some problems of a certain type
Collected in this section are miscellaneous problems about such things as what can be said of a holomorphic (or
harmonic) function when given information about how it behaves near a boundary or near innity.
Behavior near innity
1. If f(z) is an entire function which tends to innity as z tends to innity, then f(z) is a polynomial.
2. If f(z) is an injective entire function, then f(z) = az +b for some constants a and b.
3. If u(z) is a nonconstant real valued harmonic function of C, then there is a sequence z
n
C with z
n
and
u(z
n
) 0 as n .
Behavior on or near the unit circle
4. If f(z) is holomorphic in an open set containing the closed unit disk, and if f(e
i
) is real for all R, then f(z)
is constant.
5. Prove or disprove: There exists a function f(z) holomorphic on the unit disk D such that [f(z
n
)[ whenever
z
n
D and [z
n
[ 1.
6. Prove or disprove: There exists a function u(z) harmonic on the unit disk D such that [u(z
n
)[ whenever
z
n
D and [z
n
[ 1.
Other Problems
7. If f is holomorphic in the punctured disk 0 < [z[ < R and if Ref M for some constant M, then 0 is a
removable singularity.
8. If f is holomorphic in the unit disk, with [f(z)[ 1, f(0) = 0, and f(r) = f(r) for some r (0, 1),
then
[f(z)[ [z[

z
2
r
2
1 r
2
z
2

.
Solutions
1. See (1b) of April 95.
2. Suppose f H(C) is injective. Then f
1
is a continuous function in C which maps compact sets to compact
sets. Therefore, if z
n
C is any sequence tending to innity, then the image f(z
n
) cannot remain inside any
closed disk (since f
1
maps all such disks to closed bounded sets in C). Thus f(z) whenever z . By the
previous problem, f is a polynomial. Finally, if f has degree greater than one, or if f is constant, then f would not be
injective. Therefore, f is a polynomial of degree one. .
3. See (4) of April 95.
4. See (1a) of April 89.
69
2.9 Some problems of a certain type 2 COMPLEX ANALYSIS
5. & 6. That both of these statements are false is a corollary of the next two lemmas.
Lemma 1: If f H(D), then there is a sequence z
n
D with [z
n
[ 1 such that the sequence [f(z
n
)[ is
bounded.
Lemma 2: If u is harmonic in D, then there is a sequence z
n
D with [z
n
[ 1 such that the sequence u(z
n
)
is bounded.
Proof of Lemma 1: First suppose that f has innitely many zeros in D. Then, in any closed disk [z[ 1 D,
the zeros of f must be isolated (otherwise f 0). Since such a disk is compact, it contains only nitely many zeros
of f. We conclude that there must be a sequence of zeros of f tending to the boundary of D.
Nowsuppose f has nitely many zeros in D. Let
1
, . . . ,
N
be the set of all zeros of f (counting multiplicities).
Then
f(z) = (z
1
) (z
N
)g(z),
where g is holomorphic and non-zero in D. Therefore, the function 1/g is also holomorphic in D. By the maximum
modulus principal, in each compact disk D
n
= [z[ 1 1/n (n 2), the function [1/g(z)[ attains its maximum
in D
n
on the boundary at, say, the point z
n
, where [z
n
[ = 1 1/n. The reciprocals of these maxima must satisfy
[g(x
2
)[ [g(x
3
)[ . Of course, the product (z
1
) (z
N
) is bounded in D, so the sequence [f(z
n
)[ is
bounded. .
Please email comments, suggestions, and corrections to williamdemeo@gmail.com.
70
A MISCELLANEOUS DEFINITIONS AND THEOREMS
A Miscellaneous Denitions and Theorems
A.1 Real Analysis
A.1.1 Metric Spaces
The following theorem is found in Conway [3].
Theorem A.1 Let (X, d) be a metric space; then the following are equivalent statements:
(a) X is compact;
(b) Every innite set in X has a limit point (in X);
(c) X is sequentially compact
(d) X is complete and for all > 0 there exist x
1
, . . . , x
n
X such that
X =
n
_
k=1
B(x
k
, )
(The last property is called total boundedness.)
A.1.2 Measurable Functions
Denition A.1 A complex function s on a measurable space X whose range consists of only nitely many points is
called a simple function.
If
1
, . . . ,
n
are the distinct values of a simple function, and if we set A
i
= x X : s(x) =
i
, then clearly
s =
n

i=1

Ai
,
where
Ai
is the characteristic function of the set A
i
. Note that the denition assumes nothing about the sets A
i
(in
particular, they may or may not be measurable). Thus, a simple function, as dened here, is not necessarily measurable.
Continuous functions of continuous functions are continuous, and continuous functions of measurable functions
are measurable. We state this as
Theorem A.2
62
Let Y and Z be topological spaces, and let g : Y Z be continuous.
(a) If X is a topological space, if f : X Y is continuous, and if h = g f, then h : X Z is continuous.
(b) If X is a measurable space, if f : X Y is measurable, and if h = g f, then h : X Z is measurable.
Proof: If V is open in Z, then g
1
(V ) is open in Y , and h
1
(V ) = (g f)
1
(V ) = f
1
(g
1
(V )). If f is continuous, then
h
1
(V ) is open, proving (a). If f is measurable, then h
1
(V ) is measurable, proving (b).
Note, however, that measurable functions of continuous functions need not be measurable.
Theorem A.3
63
Let u and v be real measurable functions on a measurable space X, let be a continuous mapping of
the plane into a topological space Y , and dene
h(x) = (u(x), v(x)) (x X).
Then h : X Y is measurable.
62
Theorem 1.7 of Rudin [8].
63
Theorem 1.8 of Rudin [8].
71
A.1 Real Analysis A MISCELLANEOUS DEFINITIONS AND THEOREMS
A.1.3 Integration
There are a handful of results that are the most essential, and lay the foundation on which everything else is built.
Rudin [8] gives a beautifully succinct and clear presentation of these in just seven pages (pp. 2127).
64
Some of these
results are presented below, but do yourself a favor and read the master [8].
The following theorem
65
is an essential ingredient of many proofs (e.g. the proof that simple functions are dense
in L
p
, presented below).
Theorem A.4 (Rudin Theorem 1.17) Let f : X [0, ] be measurable. There exist simple measurable functions
s
n
on X such that
(a) 0 s
1
s
2
f,
(b) s
n
(x) f(x) as n , for every x X.
Theorem A.5 (Lebesgues Monotone Convergence Theorem) Let f
n
be a sequence of measurable functions on
X, and suppose that, for every x X,
(a) 0 f
1
(x) f
2
(x) ,
(b) f
n
(x) f(x) as n .
Then f is measurable, and
_
X
f
n
d
_
X
f d as n .
Theorem A.6 (Fatous lemma) If f
n
: X [0, ] (n = 1, 2, . . . ) is a sequence of positive measurable functions,
then
_
liminf f
n
liminf
_
f
n
Theorem A.7 (Lebesgues dominated convergence theorem) Let f
n
be a sequence of measurable functions on
(X, M, ) such that f
n
f a.e. If there is another sequence of measurable functions g
n
satisfying
(i) g
n
g a.e.,
(ii)
_
g
n

_
g < , and
(iii) [f
n
(x)[ g
n
(x) (x X; n = 1, 2, . . .),
then f L
1
(X, M, ),
_
f
n

_
f, and |f
n
f|
1
0.
Theorem A.8 (Egoroff) If (X, M, ) is a measure space, E M a set of nite measure, and f
n
a sequence of
measurable functions such that f
n
(x) f(x) for almost every x E, then for all > 0 there is a measurable subset
A E such that f
n
f uniformly on A and (E A) < .
64
Study these seven pages until you can recite all seven theorems and their proofs in your sleep. Also, pay attention to the details. Rudin is
careful to choose denitions and hypotheses that lend themselves to a succinct exposition, usually without too much loss of generality. For example,
he sometimes takes the range of a real-valued function to be [, ], rather than R. It is instructive to pause occasionally to consider how his
arguments depend on such choices.
65
I label it Rudin Theorem 1.17 because I cited it as such so often when practicing to take my comprehensive exams that the number stuck with
me.
72
A.1 Real Analysis A MISCELLANEOUS DEFINITIONS AND THEOREMS
A.1.4 Approximating Integrable Functions
66
Let X be a locally compact Hausdorff space. (See Rudin [8], section 2.3 for the denition.) Let C
c
(X) denote
the vector space of complex valued continuous functions on X with compact support (i.e. the closure of the set
x X : f(x) ,= 0 is compact). First, we present an important application of Rudin Theorem 1.17 (theorem A.4
above) and the dominated convergence theorem (DCT).
Theorem A.9 Let S be the set of all complex, measurable, simple functions on X such that
(x X : s(x) ,= 0) < .
If 1 p < , then S is dense in L
p
().
Proof: Let 1 p < , and f L
p
().
Case 1. f 0.
Let sn be as in Theorem 1.17. Then sn f implies sn L
p
() for all n = 1, 2, . . . . Dene gn = |f sn|
p
. Then gn L
p
(),
n = 1, 2, . . . , gn 0 as n , and gn |f|
p
L
1
(). Therefore, by the DCT,

gn 0 as n . That is,
f snp 0 as n .
Case 2. f : X [, ].
Let f = f
+
f

where f
+
= max{0, f} and f

= min{0, f}. Let sn and tn be simple functions such that


0 s1(x) s2(x) f
+
(x)
0 t1(x) t2(x) f

(x).
Then, by Case 1, we have
f
+
snp 0 and f

tnp 0
as n . Finally, note that
f (sn tn)
p
p
= f
+
f

(sn tn)
p
p
f
+
sn
p
p
+f

tn
p
p
.
Case 3. f : X C.
This case follows from Case 2 once we split f up into real and complex parts.
The following is a deep result whose proof depends on Urysohns lemma. (For the proof, see Rudin [8], section
2.4.)
Theorem A.10 (Lusins Theorem) Fix > 0. Let f be a complex measurable function which vanishes off a set of
nite measure. Then there exists g C
c
(X) such that
(x X : f(x) ,= g(x)) < .
Furthermore, we may arrange it so that
sup
x
X[g(x)[ sup
xX
[f(x)[
Lusins theorem is a key ingredient in the following. The short, elegant proof is lifted directly from Rudin. (I wont
pretend I can improve on his masterpiece.)
Theorem A.11 For 1 p < , C
c
(X) is dense in L
p
().
66
This topic is very important and appears on the comprehensive exam syllabus.
73
A.1 Real Analysis A MISCELLANEOUS DEFINITIONS AND THEOREMS
Proof: Dene S as in theorem A.9. If s S and > 0, then by Lusins Theorem there exists g Cc(X) such that g(x) = s(x)
except on a set of measure < , and |g| s. Hence,
g sp 2
1/p
s.
Since S is dense in L
p
(), this completes the proof.
We close out this subsection by giving a careful solution to a basic but important exercise from Rudin [8], Chapter
2. First, dene a step function to be a nite linear combination of characteristic functions of bounded intervals in R
1
.
(Notice how much more special these functions are than the simple functions, dened above.)
Lemma A.1 (Rudin Exercise 2.24) If f L
1
(R) then there exists a sequence g
n
of step functions on R such that
lim
n
[f g
n
[ = 0.
Proof: We proceed in steps of successively greater generality. (Ill ll in the details soon!)
Case 1. f = A for some measurable set A with (A) < .
Case 2. f =

n
i=1
iA
i
L
1
(R).
N.B. the assumption that f is integrable implies, in particular, that each Ai is measurable with (Ai) < .
Case 3. f L
1
(R), with f 0.
Case 4. f L
1
(R).
(Details coming soon!)
A.1.5 Absolute Continuity of Measures
Two excellent sources for the material appearing in this section are Rudin [8] ( 6.7, 6.10) and Folland [4] ( 3.2).
Let be a positive measure on a -algebra M, and let be an arbitrary complex measure on M. (Recall that
the range of a complex measure is a subset of C, while a positive measure takes values in [0, ]. Thus the positive
measures do not form a subclass of the complex measures.)
Suppose, for any E M, that (E) = 0 (E) = 0. In this case, we say that is absolutely continuous with
respect to , and write . If there is a set A Msuch that, for all E M, (E) = (A E), then we say that
is concentrated on A. Suppose
1
and
2
are measures on Mand suppose there exists a pair of disjoint sets A and
B such that
1
is concentrated on A and
2
is concentrated on B. Then we say that
1
and
2
are mutually singular,
and write
1

2
.
Theorem A.12 (Lebesgue-Radon-Nikodym)
67
Let be a positive -nite measure on a -algebra Min a set X, and
let be a complex measure on M.
(a) There is then a unique pair of complex measures
a
and
s
on Msuch that
=
a
+
s
,
a
,
s
.
If is positive and nite, then so are
a
and
s
.
(b) There is a unique h L
1
() such that

a
(E) =
_
E
hd E M.
67
Rudin[8], 6.10.
74
A.1 Real Analysis A MISCELLANEOUS DEFINITIONS AND THEOREMS
The pair (
a
,
s
) is called the Lebesgue decomposition of relative to . We call h the Radon-Nikodym derivative of

a
with respect to , and write h = d
a
/d and
d
a
=
d
a
d
d.
Strictly speaking, d
a
/d should be viewed as the equivalence class of functions that are equal to h -a.e.
Corollary A.1
68
Suppose is a -nite complex measure and , are -nite measures on (X, M) such that
. Then
(a) If g L
1
(), then g
d
d
L
1
() and
_
g d =
_
g
d
d
d.
(b) , and
d
d
=
d
d
d
d
-a.e.
A.1.6 Absolute Continuity of Functions
Lemma 1.2 Let f : R R be a function. If f is differentiable on [a, b], f

L
1
([a, b]), and
_
x
a
f

(t)dt =
f(x) f(a) for a x b, then f AC[a, b].
Proof Assuming the stated hypotheses, by a standard theorem,
69
f

L
1
implies that for all > 0 there is a > 0
such that, if E R is measurable mE < , then
_
E
[f

[dm < . (49)


Let A =
n
i=1
(a
i
, b
i
) be a nite union of disjoint open intervals in [a, b] such that

n
i=1
(b
i
a
i
) < . Then
mA

n
i=1
(b
i
a
i
) < , so
n

i=1
[f(b
i
) f(a
i
)[ =
n

i=1

_
bi
ai
f

dm

i=1
_
bi
ai
[f

[dm =
_
A
[f

[dm < (50)


by (49). Thus, f AC[a, b]. .
A.1.7 Product Measures and the Fubini-Tonelli Theorem
Let (X, o, ) and (Y, T , ) be measure spaces. If we want to construct a measurable space out of X Y , it is natural
to start by considering the collection of subsets o T = A B X Y : A o, B T . Note, however, that
this collection is not, in general, an algebra of sets. To get an adequate collection on which to dene product measure,
then, dene
70
o T = (o T ); that is, o T is the -algebra generated by o T .
In my opinion, the most useful version of the Fubini and Tonelli theorems is the one in Rudin [8]. It begins
by assuming only that the function f(x, y) is measurable with respect to the product -algebra o T . Then, in a
single, combined Fubini-Tonelli theorem, you get everything you need to answer any of the standard questions about
integration with respect to product measure. Here it is:
68
Folland [4], Prop. 3.9.
69
The standard theorem cited here appears often on the comprehensive exams (cf. Nov. 91 #6), but in a slightly weaker form in which the
conclusion is that |

E
f

dm| < . In the present case we need



E
|f

|dm < to get the sum in (50) to come out right.


70
This notation is not completely standard. In Aliprantis and Burkinshaw [2] (p. 154), for example, S T denotes what we call S T , while
(S T ) denotes what we have labeled S T . At the opposite extreme, I believe Rudin[8] simply takes S T to be the -algebra generated by
the sets {AB : A S, B T }.
75
A.1 Real Analysis A MISCELLANEOUS DEFINITIONS AND THEOREMS
Theorem A.13 (Fubini-Tonelli)
Assume (X, o, ) and (Y, T , ) are -nite measure spaces, and f(x, y) is a (o T )-measurable function on XY .
(a) If f(x, y) 0, and if (x) =
_
Y
f(x, y) d(y) and (y) =
_
X
f(x, y) d(x), then is o-measurable, is
T -measurable, and
_
X
d =
_
XY
f(x, y) d( ) =
_
Y
d. (51)
(b) If f : X Y C and if one of
_
Y
_
X
[f(x, y)[ d(x) d(y) < or
_
X
_
Y
[f(x, y)[ d(y) d(X) <
holds, then so does the other, and f L
1
( ).
(c) If f L
1
( ), then,
(i) for almost every x X, f(x, y) L
1
(),
(ii) for almost every y Y, f(x, y) L
1
(),
(iii) (x) =
_
Y
f d is dened almost everywhere (by (i)), moreover L
1
(),
(iv) (y) =
_
X
f d is dened almost everywhere (by (ii)), moreover L
1
(), and
(v) equation (51) holds.
76
A.2 Complex Analysis A MISCELLANEOUS DEFINITIONS AND THEOREMS
A.2 Complex Analysis
A.2.1 Cauchys Theorem
71
A continuous function : [a, b] C, where [a, b] R, is called a path in C, and such a path is called rectiable if it
is of bounded variation, i.e., if there is a constant M > 0 such that, for any partition a = t
1
< t
2
< < t
n
= b of
[a, b],

i
[(t
i
) (t
i1
)[ M. In particular, if is a piecewise smooth path, it is rectiable.
If : [a, b] C is a path in C, the set of points (t) : a t b is called the trace of . Some authors denote
this set by

, and others by . We will write

if clarity demands it. Otherwise, if we simply write z , it should


be obvious that we mean (t) = z for some a t b. Finally, if is a closed rectiable path in C, we call the region
which has as its boundary the interior of .
A curve is an equivalence class of paths that are equal modulo a change of parameter. If a path has some (non-
parametric) property that interests us (e.g., it is closed or smooth or rectiable), then invariably that property is shared
by every path in the equivalence class of parametrization of . Therefore, when parametrization has no relevance to
the discussion, we often speak of the curve , by which we mean any one of the paths that represent the curve.
Denition A.2 If is a closed rectiable curve in C then, for w /

, the number
n(; w) =
1
2i
_

dz
z w
is called the index of with respect to the point w. It is also sometimes called the winding number of around w.
Theorem A.14 (Cauchys formula, ver. 1) Let G C be an open subset of the plane and suppose f H(G). If
is a closed rectiable curve in G such that n(; w) = 0 for all w C G, then for all z G

,
f(z)n(; z) =
1
2i
_

f()
z
d.
A number of important theorems include a hypothesis like the one above concerning i.e., a closed rectiable
curve with n(; w) = 0 for all w C G (where G is some open subset of the plane). This simply means that
is contained with its interior in G. In other words, does not wind around any points in the complement of G (e.g.,
holes in G, or points exterior to G). Such a curve is called homologous to zero in G, denoted 0, and a version
of a theorem with this as one of its hypotheses may be called the homology version of the theorem.
More generally, if G C is open and
1
, . . . ,
m
are closed rectiable curves in G, then the curve =
1
+ +

m
is homologous to zero in Gprovided n(
1
, w) + +n(
m
, w) = 0 for all w CG. Thus, either theorem A.14,
or the following generalization, might be called the homology version of Cauchys formula:
Theorem A.15 (Cauchys formula, ver. 2) Let G C be an open subset of the plane and suppose f H(G). If

1
, . . . ,
m
are closed rectiable curves in G with =
1
+ +
m
0, then for all z G

,
f(z)
m

j=1
n(
j
, z) =
1
2i
m

j=1
_
j
f()
z
d.
The next theorem (or its generalization below) might be called the homology version of Cauchys theorem:
Theorem A.16 (Cauchys theorem, ver. 1) Let G C be an open set and suppose f H(G). If is a closed
rectiable curve that is homologous to zero in G, then
_

f(z)dz = 0.
71
Most of the material in this section can be found in Conway [3].
77
A.2 Complex Analysis A MISCELLANEOUS DEFINITIONS AND THEOREMS
Theorem A.17 (Cauchys theorem, ver. 2) Let G C be an open set and suppose f H(G). If
1
, . . . ,
m
are
closed rectiable curves in G such that =
1
+ +
m
0, then
m

j=1
_
j
f(z) dz = 0.
A partial converse of Cauchys theorem is the following:
Theorem A.18 Let G be an open set in the plane and f C(G, C). Suppose, for any triangular contour T G with
T 0 in G, that
_
T
f(z)dz = 0. Then f H(G).
This theorem is still valid (and occasionally easier to apply) if we replace any triangular contour with any rectan-
gular contour with sides parallel to the real and imaginary axes. This stronger version is sometimes called Moreras
theorem, and the exercise on page 81 of Sarason [10] asks you to prove it using theorem A.18.
A.2.2 Maximum Modulus Theorems
Theorem A.19 (max mod principle, ver. 1) Suppose G C is open and f H(G) attains its maximum modulus
at some point a G. Then f is constant.
That is, if there is a point a G with [f(z)[ [f(a)[ for all z G, then f is constant.
72
Theorem A.20 (max mod principle, ver. 2) If G C is open and bounded, and if f C(

G) H(G), then
max[f(z)[ : z

G = max[f(z)[ : z G.
That is, in an open and bounded region, if a holomorphic function is continuous on the boundary, then it attains its
maximum modulus there.
Theorem A.21 (max mod principle, ver. 3) Let G

C = C be open, let f H(G), and suppose there is an
M > 0 such that
lim
za
[f(z)[ M, for every a

G.
Then [f(z)[ M for all z G.
Theorem A.22 (Schwarzs lemma) Let f H(D), [f(z)[ 1 for all z D, and f(0) = 0. Then
(a) [f(z)[ [z[, for all z D,
(b) [f

(0)[ 1,
with equality in (a) for some z D 0 or equality in (b) iff f(z) = e
i
z for some constant R.
Please email comments, suggestions, and corrections to williamdemeo@gmail.com.
72
This version of the maximum modulus principle is an easy consequence of the open mapping theorem, which itself can be proved via the local
mapping theorem, which in turn can be proved using Rouch es theorem. Of course, you should know the statements of all of these theorems and,
since proving them in this sequence is not hard, you might as well know the proofs too! Two excellent references giving clear and concise proofs
are Conway [3] and Sarason [10].
78
REFERENCES
B List of Symbols
F an arbitrary eld
Q the rational numbers
Z the integers
N the natural numbers, 1, 2, . . .
C the complex numbers (a.k.a. the complex plane)

C the extended complex plane, C


R the real numbers (a.k.a. the real line)
T the unit circle, z C : [z[ = 1

R the extended real line, [, ]


Rez the real part of a complex number z C
Imz the imaginary part of a complex number z C
D or U the open unit disk, z C : [z[ < 1
H(G) the holomorphic functions on an open set G C

+
the upper half-plane, z C : Imz > 0

the lower half-plane, z C : Imz < 0


P
+
the right half-plane, z C : Rez > 0
P

the left half-plane, z C : Rez < 0

G the extended boundary of a set G



C
C[0, 1] the space of continuous real valued functions on [0, 1].
L
1
the space of integrable functions; i.e., measurable f such that
_
[f[ < .
L
p
for 0 < p < , the space of measurable functions f such that
_
[f[
p
< .
L

the space of essentially bounded measurable functions;


i.e., measurable f such that x : [f(x)[ > M has measure zero for some M < .
o T the product -algebra generated by o and T .
References
[1] Lars Ahlfors. Complex Analysis. McGraw-Hill, New York, 3rd edition, 1968.
[2] Charalambos D. Aliprantis and Owen Burkinshaw. Principles of Real Analysis. Academic Press, New York, 3rd
edition, 1998.
[3] John B. Conway. Functions of One Complex Variable I. Springer-Verlag, New York, 2nd edition, 1978.
[4] Gerald B. Folland. Real Analysis: Modern Techniques and Their Applications. John Wiley & Sons Ltd, New
York, 1999.
[5] James R. Munkres. Topology: A First Course. Prentice Hall International, Englewood Cliffs, NJ, 1975.
[6] H. L. Royden. Real Analysis. Macmillan, New York, 3rd edition, 1988.
[7] Walter Rudin. Principles of Mathematical Analysis. McGraw-Hill, New York, 3rd edition, 1976.
[8] Walter Rudin. Real and Complex Analysis. McGraw-Hill, New York, 3rd edition, 1987.
[9] Walter Rudin. Functional Analysis. McGraw-Hill, New York, second edition, 1991.
[10] Donald J. Sarason. Notes on Complex Function Theory. Henry Helson, 1994.
[11] Elias Stein and Rami Shakarchi. Complex Analysis. Princeton University Press, 2003.
79
Index
absolute continuity
of functions, 1516, 73
of measures, 6, 14, 72
approximating integrable functions, 7
area theorem, 48
Arzela-Ascoli theorem, 57, 62
Baire category theorem, 22
Banach space

p
, 35
of bounded linear operators, 26
Banach-Steinhauss theorem, 22, 35
Borel -algebra, 34
Borel set, 34
Casorati-Weierstrass theorem, 54
applied, 47, 58
Cauchys formula, 75
applied, 39, 47, 56
Cauchys theorem, 7576
converse of, see Moreras theorem
problems, 44, 45, 48, 52, 61
proof by Greens theorem, 5152
Cauchy-Riemann equations, 42, 51
closed graph theorem, 30
conformal mapping
problems, 39, 44, 50, 60, 66, 68
connected, 67
criterion for a pole, 59
curve, 75
disconnected, 67
dominated convergence theorem
applied, 11, 23, 28, 33
general version, 72
standard version, 21
Egoroffs theorem, 72
problems, 20, 2829, 3233
equicontinuity
pointwise vs. uniform, 57
equicontinuous, 62
even functions, 16
Fatous lemma, 72
Fubini-Tonelli theorem, 7374
applied, 8, 13
fundamental theorem of algebra, 52
fundamental theorem of calculus, 16
Greens theorem, 51
H olders inequality, 22
Hadamard factorization theorem
applied, 58
Hahn-Banach theorem, 22
homologous to zero, 75
implicit function theorem, 18
index, 75
inverse function theorem
of calculus, 18
Laurent expansion, 55, 59, 64
Lebesgue decomposition, 73
Liouvilles theorem
applied, 38, 48, 53
maximum modulus theorem, 76
applied, 66
monotone convergence theorem
applied, 4, 5, 12
Montels theorem, 57, 62
applied, 45
Moreras theorem, 76
problems, 48, 61
mutually singular, 72
normal family, 40, 45, 57, 6263
odd functions, 16
path, 75
Picards theorem, 38
product measures, 8, 73
Radon-Nikodym
derivative, 14, 73
problems, 78, 14, 19, 30, 34
theorem, 7273
removable singularity theorem, 59
residue theorem, 68
applied, 40, 43, 55, 65, 68
80
INDEX INDEX
Riemann mapping theorem, 60
Riesz representation theorem, 3536
applied, 1617
for L
p
, 22
Rouch es theorem, 53
Schwarzs lemma, 76
applied, 50, 61, 66, 68
Stone-Weierstrass theorem, 21
applied, 7
Tonellis theorem, see Fubini-Tonelli theorem
uniform boundedness principle, 22, 35
winding number, 75
81

Das könnte Ihnen auch gefallen